*Scarlet Fever Measles Rubella

Total Page:16

File Type:pdf, Size:1020Kb

*Scarlet Fever Measles Rubella

Crocs Scarlet fever

A 7 y.o. girl fell ill abruptly: fever, headache, severe sore throat, vomiting. Minute bright red rash appear in her reddened skin in 3 hours. It is more intensive in axillae and groin. Mucous membrane of oropharynx is hyperaemic. Greyish patches is on the tonsils. Submaxillary lymph nodes are enlarged and painful. What is your diagnosis? *Scarlet fever Measles Rubella Pseudotuberculosis Enteroviral infection

Patient R., 9 y., complains on the appearance of the erythematous precipitations, which began from the retroauricular sections, for the elongation of 72 hours they were extended to entire skin. Rashes preceded an increase in the temperature of body to 39,5°C, general weakness, cough of bronchial nature. It is known from anamnesis that 15 days ago the classmate was hospitalized into the infectious hospital. What is the diagnosis? *Scarlet fever. Measles. The chickenpox. Pseudotuberculosis. The German measles.

A 8 - year-old boy full ill abruptly fever up to 39 C, headache, recurrent vomiting, sore throat. There was punctiform rash on reddened skin on his trunk, upper part of the abdomen, and in the skin folds, especially in axillae, cubital, inguinal. The soft palate is covered with erithematous punctiform lesions and scattered petechiae. Tonsils were enlarged, reddened with superficial necrosis in the form of patches. Submaxillary lymph nodes were enlarged, dense and tender. BP is 140/90; Ps is 120 per min. What is your diagnosis? Adenoviral infection *Scarlet fever Diphtheria Measles Leukosis

A 8- year-old girl complained of fever up to 38,6 C, sore throat, rash. In medical examination lacunar tonsillitis, hyperemia and enanthema of soft palate, punctiform rash which localized mainly on flexor surface of the extremities was discovered. Nasolabial area was pale. Which antibacterial medications should be administered? Gentamycin *Penicillin Chloramphenicol Lincomycin Tetracycline

A 3-year-old child has a typical mild form of scarlet fewer and is treated at home. Which antibacterial medication should be administered for etiotropic therapy? *Erytrimycin Gentamycin Chloramphenicol Doxycycline Tetracycline

1 A 8-year-old boy has scarlet fewer and takes antipyretic and vitamins. Weakness, anorexia, skin pallor, face edema, oliguria occurs on the 14 th day of illness. Specific gravity of urine is 1030, protein level is 1 g/1, erythrocytes are 60 in the field vision. Hyaline cylinders are present too. What is complication has occurred? Tumour of urinary bladder Pyelonephritis Urolhhic disease *Glomerulonephritis Polyps of urethra

The patient of 14 years old in 2 weeks after a pharyngitis has started to complain of a ferves cence up to 38,0°С, the general delicacy, dyspnea during walking, tumescence and pain of joints of flying character. Objectively: cyanosis of lips, pulse - 100 impacts in a minute, weak filling, rhythmical. The left border of heart is displaced outside from mediaclavicular lines on 1 sm. The first tone on an apex weakened, gentle systolic hum is auscultated. What is the most probable etiological factor could cause this syndrome? Pneumococcal. Virus. Staphylococcus. *β- hemolytic streptococcus. Mushrooms.

The child of 5 years, did fall ill acutely, did increase the temperature of body to 38°C, complaint of the sore throat when swallowing, there was a disposable vomiting, appeared punctuate exanthema against the hyperemized background. A district pediatrician diagnosed it "Scarlet Fever", has appointed a hospital at home. What medication should I choose for causal treatment? *Penicillin. Tetracycline. Acyclovir. Sulfate- Polymyxin M. Interferon.

The boy of 10 years old in 2 weeks after angina had edemas of the face and moderate back pain. Objectively: a body temperature is 37,5°С, a BP - 100/80 mm hg. Urine: fresh erythrocytes are up to 100 in sight, protein - 2,20 gpl; hyaline cylinders are up to 10 in sight, relative density - 1, 020. What is the most probable diagnosis? Acute pyelonephritis. *Acute glomerulonephritis. Cancer of a kidney. Chronic glomerulonephritis. Urolithiasis.

A 10-year-old girl was admitted to a hospital with carditis presentations. It is known from the anamnesis that two weeks ago she had exacerbation of chronic tonsillitis. What is the most likely etiological factor in this case? *Streptococcus Staphylococcus Pneumococcus Klebsiella Proteus

In 3 weeks after the child has had angina, hе still has weakness, flaccidity, subfebrile temperature,

2 enlarged mandibular lymph nodes. Tonsils are quaggy, densely soldered with handles; in lacunas we can see purulent plugs. What is the most probable diagnosis? Peritonsillitis. Chronic pharyngitis. Acute lacunar angina. *Chronic tonsillitis. Tumour of tonsils.

The girl of 12 years old has arrived in cardiologic department of the hospital with manifestations of a carditis. Two weeks ago she had lacunar angina. What is the most possible etiological factor of a carditis in this case? Staphylococcus. Klebsiella. *Streptococcus. pneumococcus. Proteus.

In the child of 6 years 10 days after transferred angina on the skin of upper and lower extremities appeared symmetrically located papular - hemorrhagic rash with sizes from 3 to 5 mm in the diameter; swelling and pain in talocrural joints. In the clinical analysis of the blood: WBC. - 3,9*10*12 of/l, Hb - 124 g/l, TsP. - 0,9, thrombocytes - 250*109/l, L. - 15,0*109/l, s. - 7, 3. - 8, sg. - 70, l. - 12, m. - 3, ESR - 25 mm/h. What does be the basis of the development of hemorrhagic syndrome? Reduce the number of platelets. Reducing the concentration of plasma clotting factors. *The defeat of the vascular wall. No retraction of the blood-convolution. Violation of platelet adhesion.

Girl with chronic tonsillitis after supercooling complains on the pain in the back. Urine of the color of „ meat mud ", AD is increased to 150/100 mm Hg. In anamnesis allergic reaction to Ampicillin. What antibiotic you will appoint for the purpose of the preventive maintenance of allergic reaction? *Erythromycin. Gentamicin. Biseptol. Bitsillin (benzathine penicillin G). Ampicillin.

The child of 5 years, fell ill acutely. Doctor established diagnosis ―scarlet fever‖. Child attends children's pre-school establishment. Indicate the period of the quarantine, which is superimposed on the contact children in kindergarten. *7 days. 10 Days. 14 Days. 21 Day. 24 Days.

Masha B. 8 years old suffering from scarlet fever. Was obtained the treatment: paracetamol, vitamins. The apathy appeared on 14 days of disease, was reduced appetite, appeared pallor of the skin, pastiness and edemas of face, oliguria. Urine: specific gravity 1030, protein 1 g / l, erythrocytes 60 in the field of view, hyaline cylinders. What complications have developed in a girl? It. Pyelonephritis. Urolithiasis. Tumor-bladder.

3 *Glomerulonephritis. Polyps of the urethra.

To child of 5 years. 2 weeks it ago suffered angina. Complaints of mother on poor appetite, disrupted sleep. With an objective research is revealed heart rate 100 in 1 minute. How this index should be estimated? *Age norm. Tachycardia. Bradycardia. Respiratory arrhythmia. Paroxysmal tachycardia.

Ten days ago 6 years old girl endured angina. To the period of appeal complains on the general weakness, reduction in the appetite, two-fold vomiting, headache and pain in the back. It is objective: the skins of pale pink color, the pastiness of face, the positive symptom of Pasternatskiy. Blood: normochromic anemia, RES- of 15 mm/h, neutrophilic leukocytosis. In the analysis of the urine: protein 1,5 g/l, the lixiviated erythrocytes to 20 into the field of sight, transparent cylinders 5 in the field of sight. Your diagnosis: *Acute glomerulonephritis. Acute pyelonephritis. Chronic pyelonephritis. Chronic glomerulonephritis. Subacute malignant glomerulonephritis.

In the child of 10 years against the background of the aggravation of chronic tonsillitis on 10th the day the pain in the region of heart appeared. State of the child is of average gravity, the temperature of 37,8°C, FCR- of 93 beatings/min, weakening of the І tone above top, apical systolic noise. In the blood: leukocytes to 12*109, SES- of 28 mm/hour, the level of antistreptolysin - O of serum - 450 UP/ml, SRP - (-). On ECG: lengthening atrioventricular conductivity, a change in the final part of QRST. By the optimum version of the antibacterial therapy of myocarditis there is the designation: *Benzylpenicillin. Levomycetin. Erythromycin. Amoxycillin. Amikacin.

In boy 14 years against the background of chronic tonsillitis, maxillary sinusitis appeared the sensation of stoppages in the section of heart and additional pulse beatings. FCR- of 83 beatings/min. On ECG: after every two sinus reductions the pulses, in which the absent tooth R, QRS has duration more than 0,11 s, acutely deformed, discordant T-wave, regularly appear, complete compensating pause after which is recorded. Indicate the character of the disturbances of the rhythm: *Extrasystolia according to the type of trigeminy. Extrasystole according to the type of bigeminy. Partial A- V blockade. Complete A- V blockade. Blockade of the left leg of His's beam.

4 Pseudotuberculosis

The child of 8 years, did fall ill acutely, temperature 38,5°C, vomiting, abdominal pain. With the inspection for 3 days are revealed the yellowness of the skin and scalars, on the skin of body, brushes and feet hyperemia, punctulate rash, increase in the liver, spleen. n the eve of the disease ate salad with fresh cabbage. What is the most likely diagnosis? *Pseudotuberculosis. Viral Hepatitis. Scarlet fever. Enterovirus infection. Infectious mononucleosis.

A 10-year-old girl complained of fever, rash, abdominal pain, artralgiae on admission. In medical examination scarlet fever-like rash, symptoms of ,,hood", ,,gloves" and ,,socks" have been discovered. What is your preliminary diagnosis? Measeles Allergic rash Rubella Scarlet fewer *Pseudotuberculosis

A 5-year-old boy fell ill abruptly: fever up to 38,5, sore throat, abdominal pain. Scarlet fever-like rash, artralgiae and frequently passed stools occurred on the 4l day f illness. In medical examination hyperemia and edema of skin on the face, hands and feet, hyperemia of palatal arches, soft palate, uvula back walls of pharynx, ,,raspberry tongue", pains in epigastria and near the umbilicus was discovered. Watery stools occurred 4-6 times a day with small admixture of mucus. Padalka's symptom was positive. What's your preliminary diagnosis? Enteroviral infection Scarlet fewer Abdominal typhoid *Pseudotuberculosis Rubella

A 7-year-old boy has moderate form of pseudotuberculosis. Which antibacterial medication should be administered for etiotropic therapy? Penicillin Ampicillin *Chloramphenicol Erytromycin Cefalosporins of the first generation

A 7-year-old boy fell ill abruptly: fever up to 40 C with rigor, artralgia, myalgia, abdominal pain. Polymorphic rash with macular and scarlet fever like elements appeared on the 4th day of illness. Ictericity of skin and sclera occurred on the 6th day of illness. Liver and spleen were enlarged and tender. Padalka's symptom was positive. What's your preliminary diagnosis? Measles Rubella Scarlet fewer *Pseudotuberculosis Abdominal typhoid

5 Measles Child of 5-years. Fifth day of illness. Objective: conscious, listless. Face is puffy. Catarrhal conjunctivitis, scleritis. The skin behind the ears, on the face – there is a bright, red media papular rash, sometimes the elements of coalesce. In the throat - diffuse hyperemia, on the soft palate - enanthema. Is nott immunized. Your diagnosis? *Measles. Rubella. Scarlet fever. Pseudotuberculosis. Allergic dermatitis.

Child of 2 years old, who has arrived into the hospital with pneumonia, has nevuses pigmentosus on a skin. From anamnesis we have known that before pneumonia the child had fever and rash. What disease had the child? Rubella. *Measles. Scarlatina. ЕСНО- exanthema. Chicken pox.

There is a child of 1 year old. Against what disease there has come time to do planned immunization? *Measles. Tuberculosis. Whooping cough. Diphtheria. Poliomyelitis.

A 5-year-old boy has been ill for 5 days. The disease had abrupt onset with fever up to 38,1 C, cough, conjunctivitis, edema of eyelids. Maculopapular rash appears on his face, neck and behind his ears on the 5th day of the disease. The next day the rash spreads to his trunk. What is your diagnosis? Adenoviral infection *Measles Rubella Infectious mononucleosis Scarlet fever

A 3 year old child has been suffering from fever, cough, coryza, conjunctivitis for 4 days. He has been taking sulfadimethoxin. Today it has fever up to 39oC and maculopapular rash on its face. Except of rash the child's skin has no changes. What is your diagnosis? Scarlet fever Rubella Pseudotuberculosis *Measles Allergic rash

A child, aged 4, has being ill for 5 days, suffers from cough, skin rash, t0- 38,20С, edema of the face, photosensitivity, conjunctivitis. On the face, neck, upper part of the chest there is bright maculopapular rash with areas of merging. Hyperemic throat. Seropurulent nasal discharge. In lungs there are dry crackles. What is the most probable preliminary diagnosis? Scarlet fever Enterovirus exanthema *Measles Adenovirus infection 6 Rubella

A 7-year-old boy has been ill for 2 days. He complains of cough, coryza, headache, fever up to 38C. There is conjunctival hyperemia, epiphora, enanthema on the mucosa of the soft and in part of the hard palate, whitish papules looking like poppy-seed, surrounded by a narrow band of hyperemia opposite the molar teeth. What is your diagnosis? *Measles Rubella Scarlet fever Enteroviral infection Adenoviral infection

A child, aged 4, has being ill for 5 days, suffers from cough, skin rash, t° - 38,2°C, facial hydropy, photosensitivity, conjunctivitis. On the face, neck, upper part of the chest there is bright maculopapular rash with areas of merging. Hyperemic throat. Seropurulent nasal discharge. In lungs there are dry crackles. What is the most probable preliminary diagnosis? Adenovirus infection. Rubella. *Measles. Scarlet fever. Enterovirus exanthema.

A child is 4 years old, has been ill for 5 days. There are complaints of cough, skin rash, t-38,2ºC, face puffiness, photophobia, conjunctivitis. Objectively: there is bright, maculo-papulous, in some areas confluent rash on the face, neck, upper chest. The pharynx is hyperaemic. There are seropurulent discharges from the nose. Auscultation revealed dry rales in lungs. What is the most likely diagnosis? *Measles Adenoviral infection Scarlet fever Rubella Enterovirus exanthema

A 4-year-old child on the 5th day of illness complains of cough, rash on the skin. Temperature is 38,2°C, face is puffy, photophobia, conjunctivitis. There is a bright papulomacular rash on the face, neck, upper half of the thorax. Pharynx is hyperemiated. There are serous and purulent discharge from nose, dry rales in the lungs. What is your preliminary diagnosis? *Measles. German measles. Enterovirus infection. Adenovirus infection. Scarlet fever.

A 2-year-old boy is not vaccinated against measles. He had been in touch with the child with catarrhal period of measles and took immunoglobulin. He can fall ill within? 28 days 17 days *21 days 14 days 7 days

A 9-year-old child is ill for 5 days. Physical examination: Conscious, inert. Puffy face. Catarrhal conjunctivitis, scleritis. Bright-red papular middle-spotted skin rash on face and behind ears. Somewhere elements merge. Diffuse hyperemia in pharynx. Soft palate shows enanthema. The child is

7 not vaccinated. What is the most likely diagnosis? Allergic dermatitis. Pseudotuberculosis. *Measles. Scarlet fever. Rubella.

A 5-year-old child developed an acute disease starting from body temperature rise up to 38,5ºC, running nose, cough and conjunctivitis. On the 4th day the child presented with maculo-papular rash on face. Body temperature rose again up to 39,2ºC. Over the next few days the rash spread over the whole body and extremities. Mucous membrane of palate was hyperaemic, there was whitish deposition on cheek mucous membrane next to molars. What is your provisional diagnosis? *Measles Acute viral respiratory infection Yesinia Enterovirus diseases Rubella

Child is ill the 5th day. Disturb the temperature of body increased to 39-40°C, persistent, frequent, deep, unproductive cough, photophobia, obstruction of nose. It is objective - the puffiness of face, the mucous membrane of mouth friable, hemorrhagic enanthema before the soft palate, remainders of the spots of Filatov-Koplik’s. Your diagnosis? Scarlet fever. German measles. Adenovirus infection. *Measles. Influenza.

To child was carried out 1 year. Against what illness did come the time to conduct planned vaccination? *Measles. Tuberculosis. Whooping cough. Diphtheria. Poliomyelitis.

In the child of 5 years district doctor diagnosed measles, the first day of rash. For what period it is necessary to isolate child, if disease does flow without the complications? *Up to 5 days the rash. Up to 10-day eruption. Before the advent of pigmentation. Up to 21 days from the onset of the disease. Up to 17 days from the onset of the disease.

The child of 6 years, fell ill with crust. became ill with measles. The family has another child, 4 years old, who attends kindergarten. Concerning contact of a healthy child was put immunoglobulin. Name the period of quarantine. *21 day. 7 Days. 14 Days. 17 Days. 30 Days.

8 Rubella

A 2-year-old girl has been ill for 3 days. Today she has low-grade fever, severe catarrhal symptoms, non-abundant maculopapular rash on her buttocks and enlarged occipital glands. What is your diagnosis? *Rubella. Pseudotuberculosis. Measles. Scarlet fever. Adenoviral infection.

The two years old child has fevers up to 37,2 °С and rash on skin. Objectively: general condition is satisfactory. There is macular rash of pink color on skin of back and extension surface of extremities. Mucous tunic of palatine arches is moderate hyperemic and has fine macular rash. There are insignificant mucous discharge from a nose and enlargement of occipital lymph nodes. What is the most probable diagnosis? Enterovirus exanthema. Scarlatina. *Rubella. Chicken pox. Measles.

A 2-year-old girl has been ill for 3 days. Today she has low-grade fever, severe catarrhal symptoms, non-abundant maculopapular rash on her buttocks and enlarged occipital glands. What is your diagnosis? Adenoviral infection. Measles. *Rubella. Pseudotuberculosis. Scarlet fever.

A 2 year old girl has been ill for 3 days. Today she has low grade fever, severe catarrhal presentations, slight maculopapular rash on her buttocks and enlarged occipital lymph nodes. What is your diagnosis? *Rubella Scarlet fever Measles Adenoviral infection Pseudotuberculosis

A 5-year-old child has low-grade fever, enanthema, enlarged post-auricular cervical and suboccipital lymph nodes, pale red small macular rash, which localizes mainly on the extensor surface of the extremities, back and buttocks. What is your diagnosis? Infectious mononucleosis Measles Scarlet fever *Rubella Enteroviral infection

A 2-year-old child full ill acutely: ever and rash appeared. Now it has slight malaise, coryza and cough, enanthema on its soft palate. There is pale red small macular rash on its back and buttocks. Post-auricular cervical and subboccipitall lymph nodes are the size of a large pea, hard and slightly tender on palpation. What is your diagnosis? 9 *Rubella Measles Scarlet fever Infections mononucleosis Enteroviral infection

A baby was born on the 8 th month of gestation. It had microcephalia, cataract, cardiac defects. On the 2 nd month of the pregnancy its mother had low-grade fever, enlarged lymph nodes, and pale red small macular rash on her face, trunk and extremities. These signs disappeared after 3 days without residual occurrence. What is your diagnosis? Toxoplasmosis Cytomegaloviral infection Herpetic infection Chlamydic infection *Congenital rubella

More common congenital rubella syndrome Hydrocephaly Vesicular rush Interstitial pneumonia Thyroid disease *Hearing loss

10 Varicella

A child, aged 5, is ill with fever, vesicular rash mainly on the trunk and head skin. On the 8th day there appeared severe headache, ataxia, lethargy, movement discoordination, tremor of the extremities. On the second wave of the fever encephalitis is diagnosed. Complication of what decease can be encephalitis in this case? Herpetic infection. Enterovirus infection. Measles. *Chicken pox. Rubella.

A 5-year-old child had strong headache, vomiting, ataxia, dormancy, discoordination of movements, tremor of the extremities on the 8th day of the disease. It was followed by rise in body temperature, vesicular rash mainly on the skin of the body and the hairy part of the head. At the second wave of the fever a diagnosis of encephalitis was given. What disease complicated encephalitis in this case? *Chicken pox Measles German measles Enterovirus ifection Herpetic infection

A 7 year old girl has mild form of varicella. Headache, weakness, vertigo, tremor of her limbs, ataxia, then mental confusion appeared on the 5th day of illness. Meningeal signs are negative. Cerebrospinal fluid examination is normal. How can you explain these signs? *Encephalitis Meningitis Meningoencephalitis Myelitis Neurotoxic syndrome

On the 21 day after appearance of vesicular chickenpox rash a 7-year-old child developed ataxia, nystagmus, intention tremor, muscle hypotonia. Liquor analysis shows a low-grade lymphocytic pleocytosis, slightly increased protein rate. What complication is it? *Encephalitis Purulent meningitis Pneumonitis Acute nephritis Postherpetic neuralgia

A 9-year-old boy has been ill for 2 days. Now he has fever up to 37,5°С, maculae, papules, vesicles on his skin, skin of the head. Vesicles are round, situated on an unindurated base, surrounded by erythematous corona. What is your diagnosis? *Varicella Smallpox Scarletfewer Herpetic infection Streptodermia

A 7-year-old girl has been in contact with a patient with herpes zoster. Fever up to 39,3°С, polymorphic rash (maculae, papules, crusts, vesicles) on her skin and mucous membranes of oral cavity have occured on the 7th day of illness. What is your diagnosis? *Varicella

11 Smallpox Herpes simplex Herpes zoster Streptoderma

A 5-year-old child had strong headache, vomiting, ataxia, dormancy, discoordination of movements, tremor of the extremities on the 8th day of the disease. It was followed by rise in body temperature, vesicular rash mainly on the skin of the body and the hairy part of the head. At the second wave of the fever a diagnosis of encephalitis was given. What disease complicated encephalitis in this case? Measles. *Chicken pox. Herpetic infection. Enterovirus ifection. German measles.

The patient of 13 years old became ill acutely: T- 37,3°С, there is maculopapular, vesicular rash on a trunk, extremities, scalp. For 6-th day the condition of the patient has worsened: T - 39°С, flaccidity, headache, giddiness, unsteadiness of a gait, instability in Romberg's pose. What is your diagnosis? *Chickenpox's encephalitis. Herpetic encephalitis. Lyell's disease. Shingles. Sepsis, hematosepsis.

A 4-year-old girl has severe form of varicella. What medication should be administered except one: *Steroid hormones Acyclovir Specific immunoglobulin Recombenated interferon Inductor of endogenous interferon

A 8-year-old boy is not ill with varicella. He had been in touch with the child with varicella. He can fall ill within. 28 days 17 days *21 days 14 days 11-21 days

A 9 -year-old boy has fever, polymorphic rash - maculae, papules, vesicles and crusts on his face, skin of the head, trunk, limb and extremities. Severe headache, vomiting, ataxia, slow down and discoordination of motion have occurred on the 10th day of illness. What the complications is diagnosed? *Encephalitis Serous meningitis Neyrotoxycosis Encephalitis reaction Meningoencephalitis

A 6 -year-old girl has a lot of brown crusts on her face, trunk and extremities. What is the illness? Caposi herpetiform eczema Herpes simplex Herpes zoster

12 *Varicella Streptodermia

A 4 -year-old boy has fever, maculae, papules, vesicles and crusts on her face, skin of the head, trunk, extremities and mucous membranes of oral cavity. He has been ill for 5 days. What the time will be the patients isolated? *In 5 days after of late rash In all time of the disease In 5 days begins of the disease In 7 days after crusts In 12 days after the onset of the disease

Mum with the child of 17 years old has apply to the hospital with complaints to the general weakness, fever, morbid rash on skin of a trunk. The child is ill for 3 days. Objectively: there are grouped vesicles with is serous-muddy and hemorrhagic contents on a background of hyperemia and edema, on a lateral surface from the left side of a trunk. What is the most probable diagnosis? *Herpes Zoster. Simple contact Dermatitis. Contact allergic Dermatitis. Microbial eczema. Herpetiform During's dermatosis.

13 Infectious mononucleosis A 3-year-old girl has had fever, difficulty of nasal breathing, enlarged cervical lymph nodes. Ampicillin was prescribed by a physician. Fever up to 38,3oC, lacunae tonsillitis, maculopapular rash on the skin, enlarged liver and spleen had occurred on the 3rd day of illness. Blood count contained leucocytosis, limphomonocytosis, atypical mononuclear cells - 20 %. What is your diagnosis? Measles Rubella *Infectious mononucleosis Scarlet fever Adenoviral infection

A 10-year-old boy had complained of malaise, sore throat, difficulty of nasal breathing, fever up to 39 C on the 4th day of illness. Now he has pallor of the skin, edema of his upper eyelids, enlarged postcervical lymphnodes. Mucous membrane of oropharynx is hyperemic, tonsils are enlarged, covered by membranes, which are separated easily. Liver and spleen are enlarged. What is your preliminary diagnosis? Lymphogranulomatosis Adenoviral infection Tonsillar diphtheria Scarlet fewer *Infectious mononucleosis

A 2 year-old boy has been hospitalized with measles-like rash, tonsillitis, lymphoadenopathy, difficulty of nasal breathing and hepatosplenomegaly. What is your preliminary diagnosis? *Infectious mononucleosis Cytomegaloviral infection Measles Serum sickness Rubella

A 10-year-old child is seen in clinic and diagnosed an having infectious mononucleosis. Point to the laboratory method which helps confirm the diagnosis. Urinalysis *Blood count with atypical mononuclear cells Coagulogram Smear from oropharynx Transaminase level

A 3-year-old girl has had fever, difficulty of nasal breathing, enlarged cervical lymph nodes. Ampicillin was prescribed by a physician. Fever up to 38,3 C, lacunae tonsillitis, maculopapular rash on the skin, enlarged liver and spleen had occured on the 3rd day of illness. Blood count contained leucocytosis, limphomonocytosis, atypical mononuclear cells-20 %. What is your diagnosis? Measles Rubella *Infectious mononucleosis Scarlet fever Adenoviral infection

A 10-year-old boy had complained of malaise, sore throat, difficulty of nasal breathing, fever up to 39ºC on the 4th day of illness. Now he has pallor of the skin, edema of his upper eyelids, enlarged postcervical lymph nodes. Mucous membrane of oropharynx is hyperemic, tonsils are enlarged, covered by membranes, which are separated easily. Liver and spleen are enlarged. What is your preliminary diagnosis?

14 Lymphogranulomatosis Adenoviral infection Tonsillar diphtheria Scarlet fewer *Infectious mononucleosis

A 2 year-old boy has been hospitalized with measles-like rash, tonsillitis, lymphoadenopathy, difficulty of nasal breathing and hepatosplenomegaly. What is your preliminary diagnosis? *Infectious mononusleosis Cytomegaloviral infection Measles Serum sickness Rubella

The child of 6 years fell ill with mononucleosis. What cells of the peripheral blood do confirm the diagnosis of infectious mononucleosis? *Mononuclears. Lymphocytes. Monocytes. Neutrophils. Basophils.

A 6 -year-old child has infectious mononucleosis. Which blood cells may confirm the diagnosis? Polymorphonuclear cells Basophilic leukocytes *Atypical mononuclear cells Eosinophils Erythrocytes

Child of 5 years. Sick for day 5, the temperature of 38-39,5 ° C, sore throat when swallowing, pale, lymphopolyadenopathia. Breath snoring with his mouth open. Throat is hyperemic, tonsils are loosened with white-yellow membranes that proceeds from the lacunae. Liver is + 2.5 cm, spleen is + 2,0 cm In clinical analysis of blood there are leukocytosis, lymphocytosis, atypical mononuclear cells - 25%. How research can confirm the diagnosis? *Analysis of blood on the reaction of Paul- Bunel. Blood on sterility. Blood on blood culture. Blood on the paired sera. Examination of blood by the thick film.

A 10-year-old child is seen in clinic and diagnosed an having infectious mononucleosis. Point to the laboratory method which helps confirm the diagnosis. Urinalysis *Blood count with atypical mononuclear cells Coagulogram Smear from oropharynx Transaminase level

15 Diphtheria

A 15-years-old girl complains of weakness, sore throat, fever up to 37,9 C. She has been will 3 days after being in contact with the patient with tonsillitis. In medical examination congestive hyperemia and moderate edema of oropharyngeal mucous membrane is found. Continuous grayish membrane is the form of a film are located on the tonsils. The patches are taken off with difficulty and bleeding of the mucous membrane is present. The patches spread on to the soft palate, the palatal arches and the uvula. Regional lymph nodes are enlarged and painful. What is your diagnosis? Lacunar tonsillitis Follicular tonsillitis *Tonsillar diphteria Infections mononucleosis Simanocesky-Vincent's disease

Child of 3 years, is ill 3-rd day. Temperature 38-38,5°C, insignificant pain with the ingestion, enlarged anterior lymph nodes. With the inspection palatine almonds are swollen, the surface is covered with the whitish-grey plaque with a smooth surface, closely associated with the nearby tissues. Is established diagnosis ―diphtheria‖. What process is the basis of the formation of diphtheritic attacks? Purulent inflammation. Catarrh inflammation. *Fibrinous inflammation. Necrotic process. Dystrophic process.

The sick child complains of the general weakness, giddiness, a fevers up to 37, 5°С, pharyngalgia, edema of a neck, enlargement of submandibular lymph nodes. Objectively: mucous of guttur is hydropic and cyanotic, tonsils are enlarged, covered by membranes which extend for their limits and it's hard to take it out. What is basic mechanism of development of this disease? Action of bacterial endotoxin. Accumulation of suboxidated products. *Action of bacterial exotoxin. Allergic mechanism. Bacteriemia.

A-5-year-old boy has been ill for 3 days: low-grade fever, malaise, anorexia, dry cough, then hoarseness. Now he has striking pallor of the skin, perioral cyanosis, soundless cough, aphonia, stenotic breath, retraction of complaisant places of the thorax, tachycardia 150 strokes per min. The boy was not vaccinated. What is your preliminary diagnosis? *Laryngeal diphtheria Whooping cough Foreign body Epiglottidis Parainfluenzal laryngotracheitis

Child, 3 years, the disease began from an increase in the temperature to 37,5°C, cough, it has been hoarse voice. Not inoculated against diphtheria. On Day 3 of illness appeared short of breath, severe symptoms of respiratory insufficiency, aphonic vote. Decongestants therapy is ineffective. The most possible diagnosis? Respiratory-large. Foreign body airway. Papilomatoz larynx. *Diphtheria of the larynx. Acute Allergic laryngitis. 16 At a child, sick with diphtheria after several days developed acute myocarditis. What pathogenetic mechanism does answer for this complication? *Influence of toxin. The direct cytotoxic action of сorynebacteria. Mean molecular peptide. Products of the vital activity of bacteria. Autoimmune defeat.

A 5-year-old child has tonsillar diphtheria, which is complicated by myocarditis. What is a pathogenesis of this complication? Autoimmune lesion Cytotoxic influence of corynebacteria *Diphtheritic toxin Middlmolecular peptides Metabolic product of bacteria

A 7-year-old child is sick for 2 weeks with running nose, was taking nasal drops. The boy suffers with alimentary allergy. He applied to doctor due to suppurative and bloody discharges from nose, maceration of nostrils and upper lip. Rhinoscopy results: there are whitish-grayish areas at nasal septum. Mucous membrane of oropharynx is not changed. What is the most probable disease? Allergic rhinitis. Sinusitis (maxillar sinus)). *Diphtheria of the nose. Rhinovirus. Adenovirus.

A 7 -year-old boy has been ill for 2 weeks, when coryza occured. Vaccination terms were not kept to. Now he has difficulty of nasal breathing, sanioserous nasal discharge, excoriations on the skin at the entrance into the nose and on the upper lip. Fibrinous membranes are discovered by rhinoscopy on the mucous membrane of the nose. Mucous membrane of oropharynx is intact. What is your diagnosis? *Nasal diphtheria Adenoviral infection Rhinoviral infection Allergic rhinitis Maxillary sinusitis

A 3-year-old boy has had severe form of tonsillar diphtheria. What complication may lead to lethal outcome? Nephrosonephritis Pneumonia *Myocarditis Polyneuropathy Soft palate paralysis

A 11-year-old girl had had tonsillitis and taken erythromycin at home 3 weeks ago. Now she complains of nasal quality to the voice and nasal regurgitation .Soft palate is immovable in phonation. Vaccination terms were not kept to. What is your preliminary diagnosis? Poliomyelitis *Diphtheritic polyneuropathy Encephalitis Botulism Poliomyelitis-like form of enteroviral infection

17 The child of 6 years is ill with localized form of diphtheria of almonds. When it is possible to conduct preventive inoculations? After 1 month. *At 6 months. After 2 months. After 1 year. Vaccination not carried out.

In the child of 4 years, with the inspection are revealed pale hyperemia of almonds, their edema. On both almonds the condensed stratifications of the gray-white color, with the smooth surface, which protrude above mucous level, they are not removed by tampon. It was suspected diphtheria of the stomatopharynx. Identify priority in treating a patient. Appointment of glucocorticoids. Antibacterial therapy. Symptomatic therapy. *Introduction antidiphtheritic serum. Introduction Td toxoid.

In child at the 3rd day of disease - moderate sore throat, the subfebrile temperature of body. Dense dark gray attack covers almonds and applies to front palatine arcs and uvula. Mucous membrane in the place of defeat is hyperemized with the cyanotic nuance, is swelled. Preliminary diagnosis: extended diphtheria of the stomatopharynx. By what preparation should be conducted specific detoxication? Diphtheria toxoid. Reopolyglukine. Ringer's solution. *Antidiphtheritic antitoxic serum. 5% Glucose.

18 Mumps

Child, 5 years old, acutely ill with fever up to 37,8 ° C, headache and swelling in the parotid region. On day 2, appeared swelling in the left parotid region, tightly-elastic consistency, moderately painful on palpation. What method of research is to confirm the disease at patient? Smear-throat on flora. Blood test for sterility. Blood test for blood culture. *Wipes from the nasopharynx to the virus. Clinical analysis of blood-count with atypical mononuclear.

The child of four years old was examined by the local pediatrician. He complain to pain during a mastication and opening of a mouth, headache and fevers up to 38,5°С. Objectively: in the region of parotid salivary glands there are tumescence, moderately morbidity at a palpation, skin above it is not changed. At the examination of a gutter we can see that aperture of Stenon duct is hyperemic. What is the most probable diagnosis? Sialadenitis. Diphtheria of a guttur. Cervical lymphadenitis. *Epidemic parotitis. Infectious mononucleosis.

A 4-year-old boy complains of fewer up to 38,9 C, pain in chewing and opening the mouth, headache. Right parotis gland is enlarged and painful in palpation. The skin over the parotid glands does not change. In lesion side Stenon's duct is reddened and edematous. What is your diagnosis? Infectious mononucleosis *Mumps Rubella Cervical lymphadenitis Sialoadenitis

A 6-year-old boy complains of headache, malaise, pain in chewing, fever up to 39C.The following day the enlarged parotid glands are noticeable. Swelled parotid glands obliterate the fosses are retromaxillaris. The skin over the inflamed glands is tense and lustrous, remains of normal color. Stenon's ducts are reddened and edematous. What is your diagnosis? Tumor of parotid glands *Mumps Cervical lymphadenitis Sialolithic disease Toxic form of tonsillar diphtheria

A 7-year-old boy fell ill abruptly: fever up to 39C, headache, recurrent vomiting. Positive meningeal signs are present. The boy had been in contact with patient with mumps. He is not vaccinated against mumps. Which is more probability diagnosis? *Parotitis meningitis Meningococcal meningitis Pneumococcal meningitis Staphylococcal meningitis Tuberculosis meningitis

A 10-year-old boy had mumps. On the 5th day of illness the child's general condition becomes worse. Abdomen pain, nausea, anorexia and then vomiting occurred. Pain in the region of the left hypochondria was determined by palpation. The amylase of the blood and diastase of the urine

19 increased highly. Which lesions occurred? Appendicitis *Pancreatitis Mesadenitis Cholecystitis Gastritis

A 5-year-old boy had been ill 2 weeks ago. The disease began from dry cough, which increased gradually. Paroxysmal cough appeared and whoops occurred. In paroxysm the patient's face is read, the cervical wins are engorged. There are hemorrhages in the sclera. Leukocytes level is 16xl09/I, lymphocytes-72%, ESR-4 mm/h. What is your preliminary diagnosis? *Mumps Adenoviral infection Foreign body in respiratory tract Parainfluenza with laryngotracheitis Tuberculosis bronchodenitis

In a 10-year-old girl having mumps fever, recurrent vomiting and severe surrounding pain in upper part of her abdomen occurred on the 4th day of illness. Which is laboratory test should be prescribed to determine the diagnosis? Bilirubin level Glucose level Thymol test Activity of ALT *Analyses of blood

In the family of two children, the elder fell ill to the parotitic infection of 5 days ago. Low-order of 5 years, f mumps was not sick, attends kindergarten hat are the GP on the contact of the child? *Do not allow children to kindergarten from 11 to 21 days from the date of contact. Vaccinate against mumps. Assign antivirals. Enter immunoglobulin. Keep the garden from the moment of contact.

20 Whooping cough

Child is 1 month, suffering from whooping cough. In one of the fits of coughing the child turned blue and stopped breathing. What causes apnea? *Supraliminal inhibition of the respiratory center. Mucous coat of the larynx. Cerebral edema. Laryngospasm. Pulmonary edema.

The child of 5 years is ill for 2 weeks. First appeared the assaults of cough, then – reprise. Face of patient reddens during the cough, the veins of neck will swell. The assaults of cough conclude with vomiting. In the X-ray photograph - increased bronchial pattern. Blood tests: A - 16 g / L, lymph .- 72%, ESR - 4 mm per hour. What is the most likely diagnosis? *Pertussis. Obstructive bronchitis. Pneumonia. Adenovirus infection.

Intraorbital foreign-body.

In child, sick with whooping cough, the basic clinical manifestation of disease is paroxysmal cough. What pathogenetic mechanism does answer for this? *Forming of center of excitation in CNS according to the type of whooping dominant. Direct toxic action of agent. Inflammation of the upper respiratory tract. Formation of membranes in the upper respiratory tract. Connection of secondary flora.

Two-year-old child coughs approximately one month. For a period of recent 3 weeks the cough became attack-like. After the assault of cough - vomiting. It is objective: the puffiness of face. Sub- conjunctival hemorrhage to the right. On the skin of neck and of chest single petechiae; ulcer on the bridle of language. Your diagnosis? *Pertussis. Tubercular bronchoadenitis. Spastic bronchitis. Hemorrhagic vasculitis. RS- infection.

A 2-year-old girl complains of paroxysmal cough with whoops. There is edematous face, hemorrhages in the sclera, shallow ulcer on the frenulum of the tongue. Auscultation reveals dry rales. Examination of the oropharynx leads to cough paroxysms. What is your preliminary diagnosis? *Whooping cough Obstructive bronchitis Foreign body in respiratory tract Pneumonia Stenosis laryngotracheitis

A 2 -year- old girl had been ill for 2 weeks. Now she has paroxysmal cough with whoop xnd vomiting, hemorrhages in her sclera, shallow ulcer on the frenulum of the tongue. Auscultation reveals dry rales. Point to medication for prophylaxis of this disease: Antibiotics *APDT-vaccine ADT-anatoxin 21 ADT-M-anatoxine Bacteriophages

A 1-year-old baby has whooping cough. In one of cough paroxysms the child becomes cyanotic and its breathing disappears. Point to the cause of breathing disappearance. *Inhibition of respiratory center Edema of larynx mucous membrane. Brain edema Lung edema Laryngospasm

In kindergarten the child of 5 years fell ill to the whooping cough. Name the period of the quarantine, which is superimposed on the healthy children in the group. 7 Days 21 Day *14 days 30 Days 45 Days

22 Polimyelities

A 1,5 y.o. child fell ill acutely with high temperature 38°C, headache, fatigue. The temperature declined on the fifth day, muscular pain in the right leg occurred in the morning, there were no movements and tendon reflexes, sensitivity was reserved. What is the initial diagnosis? Viral encephalitis. Hip joint arthritis. *Polyomyelitis. Polyartropathy. Osteomyelitis.

2 years old child during 2 days had fever and insignificant catarrhal phenomena. On 4th day hi started to pull the right foot, the temperature became normalized. The doctor has suspected a poliomyelitis. What form of poliomyelitis is the most possible? *Spinal. Meningeal. Abortive. Bulbar.

Child 2 years old, desperately ill with a temperature increase up to 38,5° C, disposable vomiting, diarrhea, slight cough, runny nose. After 3 days the temperature dropped to normal, and the child not longer stand on his feet. The examination revealed limitation of movement, decreased muscle tone, absence of tendinous reflexes in the lower extremities, more in the proximal segments. Sensitivity saved. What is the most likely diagnosis? Herpetic infection. Meningococcal infection. Influenza. *Polio. Rotavirus.

Child 5,5 years old fell ill acutely with increasing temperature up to 39,2° C, repeated vomiting, coughing, runny nose. Disturbed sore throat, pain in arms and legs, in the spine, especially if he wanted to sit down. In the throat is congestion. After 2 days - there is no active movement in the lower extremities. Reduced tendon reflexes. Neck stiffness, Kernig symptom are positive. CSF: protein 0,33 g / L, cells - 65 cells in 1 ml of 65% of them - lymphocytes, 35% - neutrophils. Your diagnosis? *Polio. Viral encephalitis. Enteroviral infections, serous meningitis. Influenza. Tuberculous meningitis.

The child of 12 months old had short febrile fever, weakness and diarrhea 8 days ago. Hi has flaccidity of the right lower extremity now. The doctor suspects a poliomyelitis. What form of disease can it be? *Spinal. Meningeal. Admixed. Bulbar.

The child is ill with poliomyelitis. What term of regular medical check-up must be for the child after recovery? *For the moment of function recovery. 3 month.

23 6 months. 1 year. 2 year.

A child is 9 months old. The patient's body temperature is 36,7ºC, the skin is pale, humid, there is pain in leg muscles. There is no extremities mobility, sensitivity is present. The child has been diagnosed with poliomyelitis. The causative agent of this disease relates to the following family: *Picornavirus Paramyxovirus Tohovirus Adenovirus Rotavirus

A 1,5-year-old child fell ill acutely with high temperature 38ºC, headache, fatigue. The temperature declined on the fifth day, muscular pain in the right leg occurred in the morning, there were no movements and tendon reflexes, sensitivity was reserved. What is the initial diagnosis? *Polyomyelitis Viral encephilitis Polyartropathy Osteomyelitis Hip joint arthritis

A 3-year-old child has been taken to a pediatrician. He has no recent history of any diseases. Objective examination revealed no pathology of the internal organs. The child needs the routine immunization against the following disease: *Poliomyelitis Diphtheria and tetanus Measles, rubella, parotitis Pertussis Type B hepatitis

A 8-month-old not vaccinated baby fell ill acutely: fever up to 38,5ºC, anorexia, weakness, motor anxiety, mild catarrhal signs and diarrhea. Loss muscle tone, absence of active movement and tendon reflexes occurred on the right leg on the 5th day of illness. Cutaneous sensitivity is present. What is your preliminary diagnosis? *Polimyelities Viral encephalitis Acute respiratory viral infection Acute intestinal infection Osteomyelitis

A 2-year-old boy is having low-grade fever, mild catarrhal signs for 3 days. The body temperature decreases on the 4th day of illness and flabby paralysis on his right leg occurs. What clinical form of poliomyelitis is present? *Spinal Abortive Meningeal Pontine Bulbar

A 18-month-old not vaccinated baby was hospitalized due to flabby paralysis of the left leg. There is a loss muscle tone, absence of active movement and tendon reflexes. Cutaneous sensitivity is present. Where is the CNS lesion localized?

24 Truncus cerebri Posterior horns of spinal cord *Anterior horns of spinal cord Cerebellum Pia matter

A child is 2 years old. The disease started sharply with rise of the temperature up to 38.8°C, vomiting, headache, flaccidity, pains in the abdomen, softening of stool. In 3 days there were weakness in the left leg, hypotonia, hyporeflexia, hypodynamia at the same place, the skin on the left leg was pale and cold. What are the ways of distribution of the causative agent of this disease? Respiratory *Fecal - oral Respiratory and fecal-oral Respiratory and inoculable Fecal - oral and parenteral

A child of 8 years old is hospitalized with complaints on sharp onset of the disease, febrile fever, marked headache, repeated vomiting. On the 2-nd day of the disease the child developed nystagmus, dysphagia and aphasia. Respiration is superficial, cough is silent. What form of poliomyelitis should we think of? *Nonparalytic, a meningeal form Paralytic, a spinal form Paralytic, a bulbar form Nonapparent form Paralytic, an encephalic form

A child of 4 years old is hospitalized with complaints on sharply developed paralysis of the right leg against the background of temperature elevation up to 39°C. 4 days before there was a subfebrile fever with insignificant catarrhal symptoms, pain in the abdomen, sweating. Poliomyelitis is suspected. What symptoms, except one, will be characteristic of poliomyelitis? Atonia (hypotonia) *Adynamia (hypodynamia) Areflexia (hyporeflexia) Anesthesia (hyposthesia) Atrophy (hypotrophy)

Child is ill with poliomyelitis. On what period is established clinic observation of it after the previous disease? 3 months. 6 months. 1 years. *Until of the restoration of the lost functions. 2 years.

25 Meningococcal infection Meningitis

A 5-year-old boy fell ill abruptly: fever up to 39,5ºC, severe headache, vomiting. Meningeal signs are positive. Neutrophile pleocytosis are present in cerebrospinal fluid. Purulent meningitis is diagnosed. What is the most probable etiology of meningitis? *Meningococcus Pneumococcus Staphylococcus Hemophilus influenza Micobacterium tuberculosis

A 4-month-old-baby fell ill acutely: fever up to 39,5 C, vomiting, weakness. In 5 hours maculopapular rash occurred on the lower extremities, the buttocks, the abdomen. After 10* hours from onset of the disease hemorrhages rash then necroses appeared on the same parts of the skin. What is your diagnosis? *Meningococcal infection Rubella Measles Influenza Hemorrhagic vasculitis

A 5 year-old child fell ill sharply. Fever, intensive headache, repeated vomiting and meningeal signs dominated in the clinical picture. In 2 days he died. On autopsy: soft brain membranes were sharply sanguineous, impregnated with thick yellow-greenish exudates on the convex and basal surfaces of the brain. The brain was swollen. What disease are these changes characteristic of? *Meningococcal infection Herpetic infection, Whooping-cough Enteroviral infection Measles

The child of 4 months old became ill acutely: temperature 38,5°С. Unitary vomiting, flaccidity. In 10 hours there was rash on breech and bottom extremities in the form of petechias, maculae and papules. Some hemorrhagic elements are with a necrosis in the center. What disease can be the most probable? *Meningococcemia. Rubella. Scarlatina. Hemorrhagic vasculitis. Flu.

A 4 month old child fell seriously ill: body temperature rose up to 38,5ºC, the child became inert and had a single vomiting. 10 hours later there appeared rash over the buttocks and lower limbs in form of petechiae, spots and papules. Some hemorrhagic elements have necrosis in the centre. What is the most probable disease? *Meningococcemia Rubella Influenza Hemorrhagic vasculitis Scarlet fever

26 Child, 3 years, fell ill with the night: the temperature of the body of 40°C, repeated vomiting, headache. On the skin of body, extremities and face it appeared large sizes the hemorrhagic rash of irregular star like shape, illegible meningeal symptoms. Your diagnosis? Tuberculosis. *Meningococcemia. Enteroviral infection. Staphylococcal sepsis. Measles.

The child for 4 months old became ill acutely: temperature 38,5°С. Unitary vomiting, flaccidity. In 10 hours there was rash on breech and bottom extremities in the form of petechias, maculae and papules. Some hemorrhagic elements were with a necrosis in the center. What disease can be the most probable? *Meningococcemia. Rubella. Scarlatina. Hemorrhagic vasculitis. Flu.

Child of 6 months, is taken to the hospital in critical condition. Acutely has being ill ill last night with raising the temperature to 40 ° C, repeated vomiting, in 6 hours - appeared rash. Objectively, the 2 nd day of illness: the condition was extremely serious, dramatically listless, There is general cyanosis of the skin and mucous membranes, on the trunk, distal extremities abundant bluish color are stellar eruption from 0,5 to 1,5 cm in diameter. Muscle stiffness in the neck, tension and pulsation large fontanel are moderately expressed. Anuria. Your diagnosis? *Meningococcal infection. Staphylococcal sepsis. Hemorrhagic vasculitis. Flu, toxic form of haemorrhagic syndrome. Thrombocytopenic purpura.

The child of 4 years old became ill acutely. At examination: the patient is excited, complaints to a headache, double vomiting, body temperature is 39,2°С and there are individual petechias on the thighs and crus and expressed muscle tension of a nape, consciousness is kept, no local signs. What form of meningococcal infection has this child? *Meningitis with meningococcemia. Meningitis. Meningococcemia. Meningocephalitis. Nasopharyngitis.

The child of 5 years old has arrived into the hospital with purulent meningitis that was caused by meningococus. What kind of changes in liquor you are expecting? *Neutrophilic pleocytosis. Lymphocytic pleocytosis. Normocytosis. Hemorrhagic liquor. Lower concentration of chlorides.

A 3 year old child fell acutely ill, body temperature rose up to 39,5oC, the child became inert, there appeared recurrent vomiting, headache. Examination revealed positive meningeal symptoms, after this lumbal puncture was performed . Spinal fluid is turbid, runs out under pressure, protein concentration

27 is 1,8 g/l; Pandy reaction is +++, sugar concentration is 2,2 millimole/l, chloride concentration - 123 millimole/l, cytosis is 2,35x109 (80% of neutrophils, 20% of lymphocytes). What is the most probable diagnosis? Serous tuberculous meningitis Brain tumour Serous viral meningitis *Purulent meningitis Subarachnoid haemorrhage

1n a 10-year-old baby septic form of meningococcal infection with infective toxic shock of the 1st stage was diagnosed. Which medications should be given for start therapy? Prednisolone and immunoglobuline Prednisolone and penicillin *Levomycetin sodium succinate and prednisolone Penicillin and immunoglobulin Immunoglobulin

A 1,5 y.o. child fell seriously ill: chill, body temperature rise up to 40,10С, then rapid dropping to 36,20С, skin is covered with voluminous hemorrhagic rash and purple cyanotic spots. Extremities are cold, face features are sharpened. Diagnosis: meningococcosis, fulminant form, infection-toxic shock. What antibiotic must be used at the pre-admission stage? *Soluble Levomycetine succinate Penicillin Sulfamonometoxin Lincomycin Gentamycin

A 3 year old boy fell ill abruptly: fever up to 39ºC, weakness, vomiting. Hemorrhagic rash of various size appeared on his lower limbs within 5 hours. Meningococcemia with infective - toxic shock of the 1 degree was diagnosed. What medications should be administered? *Chloramphenicol succinate and prednisone Penicillin and prednisone Penicillin and immunoglobulin Chloramphenicol succinate and interferon Ampicillin and immunoglobulin

A child with meningococcus meningitis has been taking penicillin for 7 days. During last 4 days a body temperature is normal. Meningitic signs are absent. When is it possible to cancel antibiotic? If liquor cytosis is 100 or less, prevalence of neutrophils If leucocytosis and left neutrophil shift are absent *If liquor cytosis is 100 or less, prevalence of lymphocytes If liquor cytosis is 150, prevalence of lymphocytes Immediately

In a family, which consists of 4 persons from 3 till 30 years a cases of meningococcal infection occurs. What urgent prophylaxis should be given among contacted persons? *Antibiotics Vaccination of antimeningococcal vaccine Prophylaxis not conducted Gargling of oropharynx by antiseptics Bacteriophages

A 10-year-old boy fell ill abruptly after staying at a beach. Recurrent vomiting, severe headache, fever 28 appeared in the evening. Meningeal signs are positive. Cerebrospinal fluid is transparent, its pressure increases, the number of CSF leukocytes is 350 cells /mm3 (lymphocytes -80%), glucose level-2.21 mmol/1, protein level-0.66 g/1. Pandi's reaction (+).What is your preliminary diagnosis? Forest-spring encephalitis Purulent meningitis Tuberculous meningitis Food poisoning *Serous meningitis of viral etiology

A child is 2 years old. The disease began with rise of the temperature up to 39°C, repeated vomiting, in some hours hemorrhagic rash developed on the skin of buttocks, shins, there were spasms. On admission the condition was comatose, pulse was threadlike, 150 beats/min, arterial pressure 60/10mm Hg., plentiful confluent hemorrhagic rash on the skin of buttocks, legs, abdomen, back. What syndrome has developed in the child? Acute cardiovascular insufficiency Acute adrenal insufficiency Acute renal failure *Edema of the brain Disseminated intravascular coagulation of blood (DIC) syndrome

The child of 5 years old was ill during 1 week. The body temperature was 37, 6°С, a tumescence in the region of neck appeared. It was diagnosed: ARI, cervical lymphadenitis. Treatment: the erythromycin, hot compress on a neck. On the background of treatment the body temperature has raised up to 39, 0°С, there was headache, repeated vomiting, meningeal syndrome. What researches should be done for definition of the final diagnosis? Puncture of a cervical lymph node. General analysis of blood. Investigation of sputum for secondary flora. X-ray imaging of lungs. *Spinal puncture.

Girl of 8 years. She complains of the appearance of skin rash in a child as spots of red, up to 5 mm. The elements are arranged symmetrically, mainly in the elbow. What method can distinguish the hemorrhagic spot of vascular? *Clicking. Percussion. Grinding. Examining. Puncture.

During the inspection of the child of 1,5 years old it is established that it did fall ill acutely, it did increase temperature to 39C, appeared vomiting to 5 times. Study of the nervous system showed positive symptoms Kernig, Brudzinskogo. These symptoms relate to: *Meningeal signs. Syndrome discoordination. Syndrome of motor disorders. Reflex of the newborn. No right answer.

Child 5 years old, fell ill suddenly: a high temperature up to 39,7° C, on the thighs and buttocks appeared hemorrhagic rash star-shaped size from 0.5 to 3 cm, acrocyanosis, cold extremities, thready pulse. Doctor ambulance suspected meningococceamia. What antibiotic must be appointed to child in the prehospital stage? 29 Oxacillin. Gentamicin. *Levomitsetina succinate. Cefazolin. Rifampicin.

Children 6 years old fell ill with acute: fever up to 39,5 °C, appeared severe headache, nausea, vomiting. On examination, the child is lethargic. There are expressed rigidity of occipital muscles, positive Kernig - Brudzinskogo signs. Liquor is muddy, cells 1450 cells / mm (90% neutrophils), protein - 1.8 g / l, reaction Pandy + + +. Which antibiotic should be appointed to start therapy? *Penicillin. Ofloxacin. Tetracycline. Erythromycin. Rifampin.

Child 1 year old, is delivered into the hospital in the agonal state. It is ill the first day. Against the background temperature 40C on the skin of lower extremities there is thick hemorrhagic-necrotically rash, AD - 20/0 of mm mercury column BH - 44/min, pulse filamentary 200/min. Meningeal symptoms are negative. After 1 hour the child died, in spite of taking resuscitative measures. Is established the diagnosis of meningococcemia. Name the most possible reason for death. *Hemorrhage in the adrenal. Swelling of the brain. Acute renal failure. Acute heart failure. Acute respiratory failure.

A child 7 years old, fell ill suddenly. Fever to 39°C, headache, repeated vomiting. On examination, the patient are positive meningeal symptoms (muscle stiffness in the neck, Kernig symptom). Child 2 weeks ago contacted patients mumps infection in the classroom. Not immunized against mumps. What is the most likely diagnosis? *Mumps meningitis. Enteroviral meningitis. Meningococcal meningitis. Pneumococcal meningitis. Tuberculous meningitis.

In kindergarten to the meningococcus infection the governess fell ill, it was hospitalized. What counterepidemic measure must be taken in focus? Take the pin from the nasal wash for viruses. Conduct an active immunization. Assign antivirals. *Get contact from nasopharyngeal swab for meningococcal. Assign-antihistamines.

A 3-year-old boy suddenly fell ill with fever up to 39°C, weakness, vomiting. Hemorrhagic rash of various size appeared on his lower extremities in 5 hours. Meningococcemia with infective-toxic shock of the 1st degree was diagnosed. What medications should be prescribed? Chloramphenicol succinate and interferon. Penicillin and immunoglobulin. *Chloramphenicol succinate and prednisone. Penicillin and prednisone.

30 Ampicillin and immunoglobulin.

A 3-year-old boy fell ill abruptly: fever up to 39°C, weakness, vomiting. Hemorrhagic rash of various sizes appears on his lower limbs in 5 hours. Meningococcemia with infective – toxic shock of 1 degree was diagnosed. What medications should be administered? *Chloramphenicol succinate and prednisone. Penicillin and immunoglobulin. Chloramphenicol succinate and interferon. Penicillin and prednisone. Ampicillin and immunoglobulin.

The child of 3th years old became ill acutely, the body temperature build-up to 39,5°С, child was flaccid, had headache and repeated vomiting. At examination positive meningeal signs are appear. In the results of lumbar puncture we can see that cerebrospinal fluid is muddy, flow out under greater pressure, the contents of protein - 1,8 g/l; reaction Pandi +++, Saccharum - 2,2 millimole per liters, chlorides - 123 millimole per liters, cytosis - 2350х10/6 (80 % of neutrophils, 20 % of lymphocytes). What is the most probable diagnosis? Serous meningitis. Tumour of a brain. Subarachnoidal hemorrhage. *Purulent meningitis. Serous tubercular meningitis.

A 5-year-old boy fell ill abruptly: fever up to 39,8°C, recurrent vomiting, severe headache. Convulsions occur in 3 hours. Physician found out positive meningeal sign. Pleocytosis of 2500 cells chiefly polymorphonuclear cells, elevated protein concentration and normal glucose concentration was found in cerebrospinal fluid examination. What is your diagnosis? Tuberculous meningitis. Serous meningitis. *Purulent meningitis. Subarachnoidal hemorrhage. Encephalitis.

A 3-year-old boy fell suddenly ill: fever up to 39°C, weakness, vomiting. Hemorrhagic rash of various sizes appears on his lower limbs in 5 hours. Meningococcemia with infective - toxic shock of 1 degree was diagnosed. What medications should be administered? Penicillin and immunoglobulin. *Chloramphenicol succinate and prednisone. Chloramphenicol succinate and interferon. Ampicillin and immunoglobulin. Penicillin and prednisone.

A child of 3 years old fell ill 12 hours ago, the temperature is 39.6°C, with a headache and repeated vomiting. Objectively: languid, sleepy, hemorrhagic stellate rash on the skin of buttocks and thighs. The arterial pressure is 80/50 mm. Hg, symptoms of Kernig, Brudzinsky are expressed, there is rigidity of the occipital muscles. In liquor: cytosis 5800 cells/mm3 (80 % - neutrophils), protein 1.8 g/l. What preparation should be administered? Erythromycin *Levomycetin succinate Penicillin Gentamycin Ceftriaxon

31 A 6-year-old boy fell ill abruptly: fever up to 39,5 C, severe headache, nausea and vomiting. Meningeal signs are positive. Cerebrospinal fluid is turbid, cytosis is 1450/cells/mm3 polimorphonuclear cells-90%), protein level is 1.8 g/1, Pandi's reaction (+++) .What antibiotics should be administered for the start therapy? Tetracycline Ofloxacin *Cephalosporin Rifampicin Erytromicin

Drug of choice at the time of treatment of ІІ stages of the DIC-syndrome? *Fresh frozen plasma. Prednisolonum. Heparin. Cryoprecipitate. Kontrical.

32 Dysentery

Child 1 of year fell ill suddenly: increased the temperature of body to 39,9ºC, appeared uneasiness, vomiting, paroxysmal abdominal pain, frequent defecations up to 20 times in a 24 hour period. Defecations are liquid; contain a large quantity of mucus and the admixture of the blood. The sigmoid colon is condensed, unhealthy during the palpation. Put a preliminary diagnosis. Cholera. Staphylococcal gastroenterocolitis. *Dysentery. Salmonella. Rotavirus gastroenteritis.

A child of 1 year old was admitted to the clinic with a severe form of intestinal dysfunction, signs of neurotoxicosis. On bacterial examination Shigella of Grigoriev-Shiga. What is not characteristic of Grigoriev-Shig’s dysentery? Rapid development of colitis syndrome *Mild course Severe course Hyperthermia The expressed syndrome of infectious toxicosis

A child 3 years old is ill with a severe form of Flexner’s dysentery. What factor does not determine morphological changes in an organism and severity of dysentery? *Bacteriemia Invasive features of the causative agent An infecting dose Endo-or exotoxin formation of the causative agent Immunologic resistance of an organism

A child of 5 years old was admitted to the clinic with complaints on watery stool with mucus 5 times a day, cramp-like pains in the abdomen. On bacteriological examination Shigella Sonnei was isolated. The symptoms reflecting severity of the disease are: Character of excrements Frequency of excrements Pains in the abdomen Incompletely closed anus *All answers are true

A-5-year-old boy fell ill acutely: fever up to 38,5C, anorexia, nausea, diarrhea, stools occur 8 times daily, contain a lot of mucus and blood, thenesmus are present, sigmoid colon is tenderness and hardened. What is your diagnosis? Staphylococcal enterocolitis Salmonellosis *Dysentery Escherhiosis Viral diarrhea

A baby of 6 months is ill with a mild form of acute dysfunction of the intestine. On bacteriological examination Shigella Sonnei was isolated. What morphological changes in the intestine prevail in a similar case? *Catarrhal colitis Follicular colitis Ulcerative colitis 33 Fibrinous colitis Diphtheric colitis

The 7 years old child complains of the general weakness, spastic pain in the inferior part of a belly, mainly in the left inguinal region, frequent fluid excrements up to 18 times a day with an admixture of mucilage and blood. The beginning of disease was acutely, three days ago with a cold fit, fever and headache. The general state is medium degree of gravity; temperature is 37, 8°С. Sigmoid intestine is spastic and morbid. What is the most probable diagnosis? Amebiasis. Nonspecific ulcerative colitis. Yersiniosis. Salmonellosis. *Dysentery.

An 8-year-old boy fell ill acutely: he presents with fever, weakness, headache, abdominal pain, recurrent vomiting, then diarrhea and tenesmus. Stools occur 12 times daily, are scanty, contain a lot of mucus, pus, streaks of blood. His sigmoid gut is tender and hardened. What is your diagnosis? *Dysentery Salmonellosis Cholera Staphylococcal gastroenteritis Escherichiosis

A 8-year-old boy fell ill acutely: fever, weakness, headache, abdominal pain, recurrent vomiting, then diarrhea and tenesmus. Stools occur 12 times daily, are scanty, contain a lot of mucus, pus, streaks of blood. His sigmoid gut is tender and hardened. What is your diagnosis? *Dysentery. Escherichiosis. Salmonellosis. Cholera. Staphylococcal gastroenteritis. Tuberculin skin test.

A child of 8 years old had a fever of 38.6°С, repeated vomiting, stool 15 times with admixture of mucus and blood streaks, on examination he was pale, sigmoid intestine is spasmic. Choose optimum criteria of antibacterial therapy: *Maximal duration of 5-7 days Maximal duration is determined by dynamics of clinical manifestations In absence of the effect during 3 days it is necessary to change the antibiotic Way of introduction depends on severity of the disease and properties of the drug All answers are true

A child of 10 years old is ill with dysentery. On the 2-nd day the body temperature was 38.4°С, there were flaccidity, weakness, loss of appetite, stool was scanty, mucous, with blood, 10 times. The choice of starting antibacterial therapy in dysentery is determined by: Form of the disease Age of the child Sensitivity of shigella to antibiotics in the given area Premorbid background *All answers are true

A child of 12 years old is ill with a severe form of dysentery. Taking into consideration increase of antibiotic-resistance of the causative agent at the modern stage, it is necessary to use for starting

34 therapy of severe forms: *Cephalosporins of III generation, as a reserve – fluoroquinolones Polymixin, as a reserve - levomycetin Gentamycin, as a reserve -levomycetin Ampicillin, as a reserve - gentamycin Nifuroxazid, as a reserve - ampicillin

A baby of 10 months is ill with a severe form of acute Flexner’s dysentery. There are hyperthermia, 6 times repeated vomiting, watery stool with plenty of green mucus and blood streaks 15 times, in defecation it makes an effort, jerks its legs. What complications can arise? Infectious-toxic shock Intestinal bleeding Peritonitis, invagination Prolapse of the anus, paraproctitis *All answers are true

A child of years old is ill with an acute intestinal infection. He has a fever up to 39°С, refuses food and drinking, vomits up to 5 times, stool is frequent, watery, with admixture of mucus and greens. For bacteriological examination we use: Blood Feces and urine Lavage of the stomach and intestine Vomit *All answers are correct

A child of 8 years old has a syndrome of hemocolitis (frequent watery stool with mucus and blood, tenderness in the left iliac area) without substantial increase of the temperature. What diseases are necessary to exclude? Amebiasis Balantidiasis Chronic nonspecific inflammatory diseases of the intestine Gastrointestinal bleeding as a result of diverticulitis *All the aforesaid

35 Salmonellosis, a rotavirus infection

Child is 1 year old. Complaints: T-39,8 ° C, liquid stools. Within five days of feverish with periodic high temperature figures. Sluggish, pale. Turgor pressure was reduced. Crying without tears. Language is overlaid with a white bloom. Cardiac muted. Swollen abdomen, rumbling in the belly button, there is tenderness to palpation. Liver + 3,5 cm chair liquid, 10 times, green with slime. Your diagnosis? Dysentery. Yersiniosis. *Salmonella. Esherihioz. Rotavirus.

The boy of 11 months old after the eating of not boiled milk is ill for 5 days: temperature is 38-39°С; he has liquid excrements and vomiting. The child is weak and pallid. Tongue is furred with white deposit. Heart sounds are muffled. Belly is bloating, rumbling around the belly-button, a liver +3 cm. Excrements are liquid, up to 5 times a day, has darkly green color with an impurity of slime. What is the most probable diagnosis? *Salmonellosis. Staphylococcal intestinal infection. Rotaviral infection. Esherihiosis. Acute shigellosis.

A 10 month old boy has been ill for 5 days after consumption of unboiled milk. Body temperature is 38-39ºC, there is vomiting, liquid stool. The child is pale and inert. His tongue is covered with white deposition. Heart sounds are muffled. Abdomen is swollen, there is borborygmus in the region of umbilicus, liver is enlarged by 3 cm. Stool is liquid, dark-green, with admixtures of mucus, 5 times a day. What is the most probable diagnosis? *Salmonellosis Staphylococcal enteric infection Escherichiosis Acute shigellosis Rotaviral infection

The child of 9 months, did fall ill acutely, in winter, from an increase in the temperature to 39°C, vomiting, liquid defecations, uneasiness, catarrhal manifestations. It is hospitalized on 2 day of disease, a serious condition. Repeated vomiting. Excrements watery, discolored, unfaeces, every hour. Signs of toxic - exsicosis 2 tbsp. In laboratory examination were diagnosed as rotavirus infection. What is the main method of treatment? Probiotics. Dietotherapy. *Rehydration therapy. Enzyme preparations. Antibiotic.

The child of 7 months, became ill in the winter from an increase in the temperature to 38°C, two-fold vomiting, abundant yellow watery defecations to 10 times in a 24 hour period. What are the leading mechanism of the pathogenesis of diarrhea? *Secretory diarrhea. Invasive diarrhea. Enteroplegia. Intussusception. Anomalies of innervation of the gut. 36 A 10 y.o. child who is at oligoanuretic stage of acute renal insufficiency has got sensations of pricking in the mucous membrane of oral cavity and tongue, extremities numbness, reduced reflexes, respiratory disturbance, arrhythmia. What are these symptoms caused by? *Hyperkaliemia Hyponatremia Hyperazotemia Acidosis Alkalosis

A child of 4 years old was brought to the clinic with acute dysfunction of the intestine. Signs of enterocolitis prevail in the clinical course. On the third day blood streaks were found in feces. On the fifth day the hemolytic-uremic syndrome (Gasser’s syndrome) developed. In what escherichiosis the hemolytic-uremic syndrome is observed? Enteroinvasive *Enterohemorrhagic Enterotoxigenic Enteropathogenic All answers are true

A baby of 6 months was admitted to the clinic with acute dysfunction of the gastrointestinal tract. On bacteriological examination enteropathogenic E. Coli O111 was isolated. What affection of the gastrointestinal tract is characteristic of this causative agent? Stomach Duodenum *Small intestine Large intestine All answers are true

10 babies fell ill with a dysfunction of the intestine in the somatic postnatal department. A diarrheal syndrome prevails in the clinical picture. The most reliable method of confirmation of the diagnosis is: *Bacteriological Reaction of agglutination Reaction of indirect agglutination Reaction of passive agglutination All answers are true

12 children fell ill with a dysfunction of the intestine in the postnatal department of children's home. A diarrheal syndrome prevails in the clinical picture. What serovara of escherichia cause the disease in children more often in the first year of life? *Enteropathogenic Enterotoxigenic Enterohemorrhagic Enteroinvasive All answers are true

20 children fell ill with an acute dysfunction of the intestine in the village kindergarten. On bacteriological examination E. Coli O111 was isolated. Specify the most probable source of infection: Cattle Pigs *Human Domestic birds All answers are true

37 A child of 10 years old experiences the signs of arthritis of knee and ankle joints. What causative agent causes diarrhea with probable development of reactive arthritis? Campylobacter jejuna *Jersinia enterocolitica Salmonella Shigella All the aforesaid

A 3-month boy has been having a severe watery diarrhea for 12 hours. Objectively: tissue turgor is lowered, eyes are sunk down, the mucous membrane of the mouth is dry, oliguria. What causative agents these symptoms are characteristic of: Shigella dysenteriae Enterohemorrhagic E. Coli Salmonella typhi. Enteroinvasive E. Coli *Enterotoxigenic E. Coli, Vibrio Cholerae Helicobacter pylori, Plesiomonas shigeloides

A child fell abruptly ill. Frequent watery stool developed 6 hours ago. The body temperature is normal. Then vomiting took place. On physical examination: child 's voice is hoarse, eyes are deeply set in the orbits. The pulse is frequent. Blood pressure is low. There is no urine. What is the preliminary diagnosis? Salmonellosis Dysentery *Cholera Toxic food-borne infection Typhoid fever

Sick M., 2 years, for the third day is found on the treatment apropos of acute enterocolitis. Antibacterial and detoxification therapy is obtained. State without the worsening. It is objective: the general state is severe, consciousness is preserved, but darkened. The skin is pale. Pasty, AP -90/60 mm Hg, FCR- of 132 beatings/min, RFR - of 34/min. In the analyses of the blood: Hb of 70 g/l, Er - 2,3x1012/l, general. protein 58 g/l, general. bilirubin 18,6 mmole/liter: direct 4,8 mmole/liter, urea 24 mmole/liter, creatinine -140 of mkmole/l. Diuresis 200 ml/day. About what complication of main disease it is necessary to think? Hemolytic- uremic syndrome. Infectious-toxic shock. *Acute kidney deficiency. Anhydremical shock. Septic shock.

Sick A., 3 years, are found on the treatment in the resuscitation department with the diagnosis: Acute kidney deficiency, oligoanuric stage. On ECG: high T-wave, the expansion of complex QRS, the displacement of the interval S- T lower than contour isoline. About what disturbance of electrolytic balance it is possible to think? *Hyperkaliemias. Hypokalemias. Hypocalcemias. Hypercalcemias. Hyperphosphatemia.

A baby, aged 2 years, has a syndrome of malabsorbtion. On ECG: S-T segment depression, wave T inversion and high wave U. What is the cause of these changes? Hypomagnesaemia.

38 Hyperkalihistia. Hypercalcihistia. *Hypokalihistia. Hypocalcihistia.

At the child of 2 years old with malabsorption syndrome on electrocardiogram was detect depression of ST-T segment, inversion of T wave and increase of U wave. What does this changes caused by? *Hypokaliemia. Hyperkaliemia. Hypercalcemia. Hypocalcemia. Hypomagnesemia.

The girl of 7 years old is in intensive care department with acute renal insufficiency. In the biochemical analysis of blood the level of potassium is 7 mmol/L. This condition is menacing on origin of: *Asystole. Anorexia. Anuria. Hypoxemia. Hypercapnia.

The girl of 7 years, is situated in the intensive care unit apropos of acute kidney deficiency. In the biochemical analysis of the blood the level of potassium is 7 mmole/liter. This state threatening on the appearance: Anorexia Anuria. Hypoxemias. *Asystolia. Hypercapnia.

During the first days of the disease at an acute intestinal infection gastroenterocolitis, child 9 months lost 5% of body weight. How can we eliminate the water - the salt scarcity? *Assign oral rehydration. Assign parenteral rehydration for 2 days. Assign a rational diet therapy. Assign enteros-sorbent. Assign biopreparations.

39 Viral hepatits Jaundice

With the inspection of children from the focus of virus hepatite A in the 8-years old child in the absence of clinical manifestations in the blood they are revealed: anti- -HAV - Ig M in the high concentration, ALT-1,8 mmole/liter. Your diagnosis? Hepatitis B. Hepatitis C. Hepatitis D. *Hepatitis A. Hepatitis E.

In the child of 7 years the preliminary diagnosis of acute virus hepatite A was established. What research will confirm the etiology of disease and the acuteness of process? Anti- -HAV-IgG, by the method of IFA. Anti- -HAV-IgM, by the method of IFA. *Anti- -HAV-IgA, by the method of IFA. Anti- -HAV-IgE, by the method of IFA. Anti- -HAV-IgD, by the method of IFA.

The boy of 10 years old has chronic virus hepatitis B with the maximal activity. What of this laboratory tests can the most precisely characterizes degree of cytolysis? Crude protein. Takara-Ara's test. Prothrombin. *Transaminases. Veltmam's test.

The child of 8 years old has clinic of hepatitis. At the examination HBsAg, antibodies of Ig M class to korovsky antigen were detect. What hepatitis has the child? *Virus hepatitis B. Virus hepatitis C. Virus hepatitis D. Virus hepatitis Е. Virus hepatitis A.

At the child of 3 months old icterus and hepatosplenomegaly has appeared after several days of anxiety, anorexia, low grade fever; urine has dark color, an excrement are decolorized. In the age of 1 month there were hemotransfusions. What is the most probable diagnosis? Hemolytic anemia. Biliary atresia. *Virus hepatitis B. Conjugation jaundice. Virus hepatitis A.

In 3-month-old baby with low-grade fever, motor anxiety, anorexia, jaundice, hepatosplenomegaly, dark urine, discolored stool occurs. The baby has had blood transfusion 2 month ago. What is the most probable diagnosis? *Viral hepatits B Hemolytic anemia Viral hepatitis A Atresia of biliferous tructs Conjugation jaundice

40 A 10-year-old child is sick with chronic viral hepatitis B with marked activity of the process. Total bilirubin — 70 µmol/L, direct – 26 µmol/L, indirect 44 µmol/L, AST - 6,2 mmol/L, ALT - 4,8 mmol/L. What mechanism underlies the transaminase level increase of this patient? Intrahepatic cholestasis. Hypersplenism. Failure of the synthetic function of the liver. *Cytolysis of hepatocytes. Failure of bilirubin conjugation.

A 6-year-old boy is seen in clinic and diagnosed as having mild form of viral hepatitis A. What is the most important in treatment? *Regimen and diet Hepatoprotectors Antiviral medications Corucosteroides Infusion therapy

At boy 10 years old yellow skin and sclera, abdominal pain, nausea, increased liver and spleen appeared. Previously, due to blunt abdominal trauma was made a blood transfusion. Total bilirubin 42.0 mmol / L, direct - 26 micromol / l, indirect - 12 micromol / l, AST - 4,2 mmol / L, ALT - 3,6 mmol / l were detected RNA and antibody Ig class M hepatitis B virus C. Enter a rational treatment in this phase of the disease. Hemosorbtion. Glucocorticoids. Infusion Therapy. *Antivirals. Intestinal chelators.

The boy of 8 years year ago was injured with hepatitis B. In the recent two months he complains about the increased fatigue, the disturbance of sleep, worsening in the appetite, nausea, especially in the morning. Integuments without jaundice, liver and spleen are palpated 1 cm 1 cm below the costal edge, painless. Activity of ALT is 2.2 mmol / liter. This state can be estimated as: Relapse-hepatitis B. Dyskinesia of biliary-tract. Residual effects of endured hepatitis. *The development of chronic hepatitis. The development of cirrhosis.

Children of 3 months. He has been restless during the week, regurgitate, refused food, the body temperature is subfebrile. Two days ago, appeared dark urine and discolored feces, then - jaundice. Liver and spleen were enlarged. In the neonatal period the child received a blood transfusion. What is the most likely diagnosis? *Viral hepatitis C. Viral hepatitis A. Hemolytic anemia. Conjugate jaundice. Atresia biliary tract.

4-year-old patient was admitted to the intensive care unit with hemorrhagic shock due to gastric bleeding. He has a history of hepatitis B during the last 5 years. The source of bleeding is esophageal veins. What is the most effective method for control of the bleeding? Intravenous administration of pituitrin. Administration of plasma.

41 Operation. *Introduction of obturator nasogastric tube. Hemostatic therapy.

Jaundice

The skin of 3 days old child has got yellow color. The child was born with weight 3.200 kg, length of a body 52 sm. active. There is puerile respiration above lungs. Breathing rate - 36 in 1 minute, heart sounds are rhythmical. The heart rate - 130 impacts in a minute, Belly soft. The liver + 2 sm, lien is not palpated. Excrements in the form of meconium. What is the most probable diagnosis? Hemorrhagic disease of newborn. Sepsis of newborns. *Physiological icterus. Atresia of the bile passages. Minkowsky-Shauffard disease.

The child has a 3 day life has become icteric skin color. Child was born with a weight of 3,200 kg, body length 52 cm Active. Above Light puerilnoe breath. BH - 36 in 1 min. Cardiac rhythmic. Heart rate - 130 in 1 min. Abdomen soft. The liver appears from under the costal arch to 2 cm spleen was not palpiruetsya. Excretions are in the form of meconium. The most likely diagnosis: *Pedicterus. Hemolytic disease of newborn. Neonatal sepsis. Anemia- Minkowski-Shafar. Atresia of bile duct.

In reported newborn from 3 through 10 days of life jaundice was observed. The general condition remained satisfactory. The maximum level of bilirubin in the blood at that time was 102 mmol/l, of which 8.2 mmol/l at the expense nonconjugated. Development of a what state is most probably this child? Fetal-hepatitis. Hemolytic disease of newborn. *Pedicterus. Hereditary icterohemolytic anemia. Atresia, biliary tract.

The child of six months old since the birth has icterus with greenish shade. The manifestations of hemorrhagic diathesis and dermal itch are observed. What is the most probable pathology at the child? Gilbert's syndrome. Dabin-Johnson's syndrome. *Biliary atresia. Crigler-Najjar syndrome. Erythroblastosis.

A neonate was born from the 1st gestation on term. The jaundice was revealed on the 2nd day of life, then it became more acute. The adynamia, vomiting and hepatomegaly were observed. Indirect bilirubin level was 275 mol/L, direct bilirubin level - 5 mol/L, Hb - 150 g/l. Mother’s blood group - 0[I], Rh+, child’s blood group- A[II], Rh+. What is the most probable diagnosis? *Hemolytic disease of the neonate [АВО incompatibility], icteric type Jaundice due to conjugation disorder Hepatitis Physiological jaundice

42 Hemolytic disease of the neonate [Rh - incompatibility]

A baby boy was born in time, it was his mother's 1st pregnancy. The jaundice was revealed on the 2nd day of life, then it progressed. The adynamia, vomiting and hepatomegaly were presented. The indirect bilirubin level was 275 mcmol/L, the direct bilirubin level - 5 mcmol/L, Hb- 150 g/L. Mother's blood group - 0(I), Rh+, child's blood group - A(II), Rh+. Make a diagnosis. *Hemolytic disease of newborn (АВО incompatibility), icteric type Jaundice due to conjugation disorder Hepatitis Physiological jaundice Hemolytic disease of newborn (Rh - incompatibility)

A full-term baby (the 1st uncomplicated pregnancy, difficult labour) had a cephalogematoma. On the 2nd day there was jaundice, on the third the following changes in neurological status appeared: nystagmus, Graefe syndrome. Urine was yellow, feces were of golden-yellow colour. Mother's blood group is A (II) Rh-, the baby's one - A (II) Rh+. On the third day the child's Hb was 200g/l, RBCs - 6,1\cdot10^{12}/l, blood bilirubin - 58 micromole/l at the expense of unbound fraction. What caused the jaundice in the child? *Craniocerebral birth trauma Physiological jaundice Neonatal anaemia Biliary atresia Fetal hepatitis

43 Flu

A child of 3 years old had sharply elevated body temperature, there was a repeated vomiting, the child was restless. On examination: mucous membrane of the fauces was hyperemic, there were no coatings. The tremor of the extremities, increased tendon reflexes, rigidity of the occipital muscles were marked. In liquor: cytosis - separate cells, protein - 0.33g/l. On the 4-th day the condition improved, meningeal signs disappeared. What is your diagnosis? Tuberculous meningitis Poliomyelitis, meningeal form Encephalitis of viral etiology Enteroviral meningitis *Flu, meningism

A 3 y.o. girl has had a temperature rise up to $38^0С$, rhinitis, dry superficial cough, flabbiness, appetite loss. Palpation didn't reveal any changes over her lungs. Percussion sound has a wooden resonance, auscultation revealed puerile breathing, no rales. In blood: leukopenia, lymphocytosis, increased ESR. What is the most probable diagnosis? *Acute simple tracheitis Acute obstructive bronchitis Recurrent bronchitis, acute condition Acute simple bronchitis Bilateral microfocal pneumonia

Ambulace brought to the hospital a patient with acute respiratory viral infection. The illness began suddenly with temperature rise up to 39,90С. He complains of headache in frontotemporal lobes, pain in eyeballs, aching of the whole body, nose stuffiness, sore throat, dry cough. At home he had a nasal hemorrhage twice. What type of acute respiratory viral infection is it? *Influenza Adenoviral infection Parainfluenza RS- infection Enterovirus infection

A patient, aged 16, complains of headache, mainly in the frontal and temporal areas, superciliary arch, appearing of vomiting at the peak of headache, pain during the eyeballs movement, joint's pain. On examination: excited, t0- 390С, Ps- 110/min. Tonic and clonus cramps. Uncertain meningeal signs. What is the most likely diagnosis? *Influenza with cerebral edema manifestations Adenovirus infection Respiratory syncytial virus Influenza, typical disease duration Parainfluenza

Child is 5 days. From І full-term pregnancy in the second half of which my mother suffered a flu. On examination, marked reduction of motor activity, pale skin, and periorbitalny perioral cyanosis, which is reinforced by sucking the breast, anxiety, ―puffing‖, the retraction of inter-rib spaces. Percussion revealed an increase in relative cardiac dullness borders on all sides, auscultatory-tachycardia, heart rate of 170 in 1 minute, heart tones are weakened. Tachypnea. Hepatomegaly. On x-ray of the chest there is increased the size of the heart. The ECG are recorded signs of myocardial hypertrophy of the ventricles, persistent violations of rhythm and conduction. Which of the following is the most likely cause of such changes? *Non-rheumatic myocarditis. Fibroelastoz.

44 Ductus arteriosus. Constrictive myopericarditis. Infective endocarditis?

The patient of 10 years, in whom on 2nd day of disease are observed the symptoms of the influenza of average gravity, obtains aspirin, gluconate of calcium; naphthazoline 0,1% in the drops into the nose. What antiviral preparation it is expedient to appoint? *Remantadin. Laferon. Acyclovir. Ribavirin. Herpevir.

Parainfluenza virus

A child is 2 years old. The child complains of hoarse voice, dyspnoea with obstructed inspiration. The disease started 3 days ago from dry cough and nose stuffiness. Objectively: general condition is unbalanced, stridor is present. The child's skin is pale. Body temperature is 37,7ºC. The palatine arches are hyperemic. There is no deposit. Heart sounds are rhythmic. Auscultation of lungs reveals rough breathing sounds, crepitation is absent. Parainfluenza virus has been detected in nasopharynx lavage. What is the most likely diagnosis? *Acute laryngotracheitis Epiglottitis Foreign body Diphtheria Laryngospasm

The girl of 4th years old became ill suddenly. The voice became hoarse, sometimes aphonic. There was coarse, sonorous, dry, unproductive tussis. Body temperature was 37,4°С. Objectively: the child is quiet; position in bed is compelled (with the fixed shoulder girdle). Stenotic respiration is well audible on distance. It is precisely visible a permanent job of respiratory musculation. Auscultation: respiration is rigid, even depressed, especially in back and bottom departments of lungs. What mechanism dominates over a pathogenesis of a croup? *Edema of a mucosa of larynx and trachea. Spasm of unstriated muscles of larynx and trachea. Obstruction. Laryngospasm. Hypersecretion of glands of mucosa of a larynx, trachea and bronchi.

An infant aged 1 year on the third day of common cold at night developed inspiratory stridor, hoarse voice and barking cough. Physical examination revealed suprasternal and intercostals chest retractions. There is a bluish skin discoloration. Moistly seen over the upper lip. The respiratory rate is 52 per min and pulse - 122 bpm. The body temperature is 37,5°C. What disease does the infant have? Acute bronchiolitis with respiratory distress. Bronchopneumonia without complications. Acute epiglottitis. *Acute infectious croup due to viral laryngotracheitis. Acute laryngitis.

Child of 10 months. It fell ill acutely: increased the temperature of body to 39ºC, appeared cough and runny nose. On day 2 of disease at night the child suddenly became restless, appeared the rough,

45 ―barking‖ cough, hoarseness of voice, inspiratory shortness of breath. About what illness it is possible to think? Diphtheria. Pertussis. Intraorbital foreign- body. *Paragrip. TB bronhoadenit.

A 1 year-old baby fell ill abruptly fever up to 38ºC, coryza, cough. Hoarseness of voice, dry barking cough, stenotic breathing and motor anxiety occur at night. What pathogen caused this disease? Adenovirus Corynebacterium diphtheria *Parainfluenza virus Rhinovirus Reovirus

A 9-month-old baby has spinal form of poliomyelitis. What group of viruses does the causative agent of the disease belong to? *Picornaviruses Paramyxoviruses Togoviruses Adenoviruses Rotaviruses

A baby of 9 months fell ill sharply: the temperature rose up to 38.8°C, there was a cough, hoarse voice. In some hours the condition sharply aggravated, symptoms of laryngotracheitis with stenosis of I-II degree developed. What of causative agents has most likely caused croupe in the baby? Virus of flu *Virus of paraflu Adenovirus Enterovirus Rhinovirus

Child 2,5 years old, fell ill with acute illness started with a barking cough, moderate secretions from the nose, temperature 38°C. In the first day of the disease the child awoke at night because of rough cough, shortness of breath. Shortness of breath is of inspiratory nature, is observed as the excitation, and in rest. Breathing involved supporting musculature. Is noted the blurry retraction of the yielding places of chest and epigastrium, perioral cyanosis, pallor, and tachycardia. The child immunized by age. What is the best possible preliminary diagnosis? *ARI, acute laryngotracheobronchitis, stenosis of 2 degree. Diphtheria-respiratory. Intraorbital foreign-body airway. Asthma. TB intrathoracic lymph nodes.

46 Acute respiratory viral infection and complications

The child of 14 years old has complained of weakness, temperature rise up to 37,8°С, mucous discharge from a nose, and pharyngalgia during swallowing, feeling of "sand" in eyes during 7 days. Objectively: the enlargement of backcervical and submandibular lymph nodes, edema and injection of vessels of conjunctivas, hyperemia of mucosa of guttur, hypertrophy of tonsils, no incrustation. What is the most probable diagnosis? Flu. Parainfluenza. Infectious mononucleosis. *Adenoviral infection. Rhinoviral infection.

A 3 m.o. child fell seriously ill, body temperature rised up to $37,8^0C$, there is semicough. On the 3- rd day the cough grew worse, dyspnea appeared. On percussion: tympanic sound above lungs, on auscultation: a lot of fine moist and wheezing rales during expiration. What is the most probable diagnosis? *Acute respiratory viral infection, bronchiolitis Acute respiratory viral infection, bronchopneumonia Acute respiratory viral infection, bronchitis Acute respiratory viral infection, bronchitis with asthmatic component Acute respiratory viral infection, focal pneumonia

A 3 month old infant suffering from acute segmental pneumonia has dyspnea (respiration rate - 80 per minute), paradoxical breathing, tachycardia, total cyanosis. Respiration and pulse - ratio is 1:2. The heart dullness under normal size. Such signs characterise: *Respiratory failure of III degree Respiratory failure of I degree Respiratory failure of II degree Myocarditis Congenital heart malformation

A child was born at a gestational age of 34 weeks in grave condition. The leading symptoms were respiratory distress symptoms, namely sonorous and prolonged expiration, involving additional muscles into respiratory process. The Silverman score at birth was 0 points, in 3 hours it was 3 points with clinical findings. Which diagnostic study will allow to diagnose the form of pneumopathy? *X-ray of chest Clinical blood test Determination of blood gas composition Proteinogram Immunoassay

The 10 y.o. boy has complains on headache, weakness, fever 40ºС, vomiting, expressed dyspnea, pale skin with flush on right cheek, lag of right hemithorax respiratory movement, dullness on percussion over low lobe of right lung, weakness of vesicular respiration in this zone. The abdomen is painless and soft at palpation. Which disease lead to these symptoms and signs? *Pneumonia croupousa Intestinal infection Acute appendicitis Acute cholecystitis Flu

An 18-month-old child was taken to a hospital on the 4-th day of the disease. The disease began

47 acutely with temperature 39, weakness, cough, breathlessness. He is pale, cyanotic, has had febrile temperature for over 3 days. There are crepitative fine bubbling rales on auscultation. Percussion sound is shortened in the right infrascapular region. X-ray picture shows non-homogeneous segment infiltration 8-10 mm on the right, the intensification of lung pattern. Your diagnosis: *Segmentary pneumonia Grippe Bronchitis Bronchiolitis Interstitial pneumonia

A 6 week old child is admitted because of tachypnea. Birth had been uneventful, although conjunctivitis developed on the third day of life and lasted for about 2 weeks. Physical examination reveals tachypnea, bilateral inspiratory crackles and single expiratory wheezing. Bilateral pneumonia is evident on chest X-ray. The child is afebrile and has no history of fever. White blood cell count is 15cdot109/l, with 28% of eosinophils. The most likely cause of this child's symptoms is: *Clamydia trachomanis Pneumocystis carinii Mycoplasma pneumoniae Visceral larva migrans Varicella

A 4 year old girl was playing with her toys and suddenly she got an attack of cough, dyspnea. Objectively: respiration rate - 45/min, heart rate - 130/min. Percussion revealed dullness of percussion sound on the right in the lower parts. Auscultation revealed diminished breath sounds with bronchial resonance on the right. X-ray picture showed shadowing of the lower part of lungs on the right. Blood analysis revealed no signs of inflammation. The child was diagnosed with foreign body in the right bronchus. What complication caused such clinical presentations? *Atelectasis Emphysema Pneumothorax Bronchitis Pneumonia

A 3-year-old child was playing in a playpen when he suddenly developed paroxysmal cough and shortness of breath. Objectively: dry cough, mixed dyspnoea. Lung auscultation revealed some wheezes. Breathing sounds on the right are diminished. The child doesn't mix with other children. Immunization is age-appropriate. What pathological condition can be suspected? *Foreign body in the respiratory tracts Pneumonia Acute respiratory viral infection Pertussis Bronchial asthma

The 7 m.o. infant is suffering from acute pneumonia which was complicated by cardiovascular insufficiency and respiratory failure of II degree. The accompanied diagnosis is malnutrition of II degree. Choose the best variant of therapy: *Ampiox and Amicacin Macropen and Penicillin Penicillin and Ampiox Gentamycin and Macropen Ampiox and Polymixin

48 A baby, aged 8 months, is examined and a diagnosis of atypical community-acquired pneumonia of chlamidial etiology is made. What is the optimum alternative of antibiotic therapy in this case? Aminopenicillin. Cephalosporin of the II generation. *Macrolide antibiotic of the II generation. Aminoglycoside antibiotic. Macrolide antibiotic of the I generation.

A 8-month-old child is diagnosed atypical community-acquired Chlamidia trachomatis pneumonia. Select the best antibiotic. Macrolide of 1st generation. Aminopenicilline. *Macrolide of 2nd generation. Cephalosporine of 2nd generation. Aminoglycoside.

The child of 8 months old after corresponding examination has such diagnosis as atypical community- acquired pneumonia of clamydion etiologies. Choose an optimum variant of an antibioticotherapia: Cephalosporin of second generation. Aminoglycoside. Aminoglycosides. *Macrolide of second generations. Macrolide of first generations.

A 1-year-old infant is admitted for failure to thrive. During the neonatal period he had an exploratory laparotomy for intestinal obstruction. At 3,8 and 11 month of age, he had respiratory infections diagnosed as bronchitis. Physical examination: weight of 6,8 kg, thin extremities with very little subcutaneous tissue, and a protuberant abdomen. The essentials diagnostic study in this child is: Skin test for milk allergy. Bronchoscopy. *Sweat electrolytes. Serum immunoglobulin le

In child with the hereditary hemolytic anemia against the background of ARD, high fever the ictericity of the skins and mucosa appeared, increased the liver and spleen. What complication of hemolytic anemia is threatening for the life of child? Bilirubinic encephalopathy. Hepatic insufficiency. Heart insufficiency. *Haemolytically-uremic syndrome. Hyperthermal syndrome. he child is 11 years. The complaints of general weakness, rapid fatigue, sweating, anorexia, weight loss, presence of rash, pain in the heart, joints, muscles, bones, increased body temperature to subfebrile digits, with short-term rises to 39-40 ° C daytime or evening with a fever and subsequent perspire (spreads of temperature during the day more than 1,5 ° C). On examination, there is pale skin with a grayish tinge. At sites shins, forearms, neck, elbow, lateral surfaces of the trunk, oral mucosa, transitional age folds is visible small hemorrhagic rash. Finite phalanx of fingers are in the form of drum sticks, nail-type time stack. Auscultation of the heart auscultated protodiastolic "sawing" noise in the IV intercostal space near the left edge of the sternum, which is worse on inspiration. Which of the following diseases most likely have a child? Rheumatism. Congenital carditis. 49 Non-rheumatic carditis. *Infective endocarditis. Systemic lupus erythematosus.

In the 3-year-old child after AVR deteriorated general condition, appeared quick fatigue. Skin pale, enlarged left heart border, I tone is deaf on the top, soft systolic sound over the top. On ECG signs of left ventricular overload. What is the most likely diagnosis? *Non-rheumatic myocarditis. Rheumatic carditis. Congestive Cardiomyopathy. Congenital carditis. Acquired heart disease.

Girl 9 years old in 2 months of age suffered pneumonia, in 3 years with severe measles. Underwent a serious flu with high fever. Within 3 days against the backdrop of fever felt fear, was restless, saw devils "with a nasty snouts, which something spoke. After lowering the temperature and improve well- being on day 7 went to school, but quickly tired, headache, 4 days ago saw on the wall to wall pictures snakes, cockroaches, there were "voices" again became restless and therefore was hospitalized in psychiatric hospital. What is it need to be assigned a patient to prevent re-delirious state of infectious origin? Detoxification therapy. Antidepressant. Restorative therapy. *Tranquilizers. Nootropics

A child 12 years diagnosed with non-rheumatic carditis, acute, moderately severe, with signs of CI of I degree in 7 days after acute respiratory viral infection. What is the pathogenetic mechanism is the basis of the disease? Infection. Allergic. Autoimmune. *Infectious-allergic. Toxic.

In 10 years old girl after ARVI the complaints of pain in the heart and shortness of breath at the physical load appeared. During inspection: the skin is pale, the increased humidity. The left measure of heart is displaced to the mediaclavicular line, heart tones are weakened, there is tender systolic noise above the top. FCR-124 per minute, AP-90/60 mm Hg. Blood: anemia of I-st grade, moderate leukocytosis, eosinophilia, the proteins of acute phase are in norm. ECG: sinus tachycardia, the disturbance of the processes of repolarization, the displacement of the interval ST lower than contour isoline. The described symptoms are characteristic for: *Nonrheumatic carditis. Pancarditis. Septic endocarditis. Fibroelastosis. Rheumocarditis.

In a family of 5 persons (3 children) there is sick child of 11 months. Appeared lethargic, then raised the body temperature to 39 ° C, then - nasal congestion, repeated vomiting. Objective: a serious condition, and occasional hemorrhage in the neck, palate mucosa, sclera. Throat is cyanotic, there is grainity o posterior wall of the pharynx. From the nose - scarce mucous discharge. Breathing is hard, 46 per minute. Pulse - 156 per minute. Cardiac is muted. What is the primary measure is held in the

50 hearth? *Isolation patient in Isolation unit. Hospitalization of the patient in a somatic department. The final disinfection. Antibiotic. Introduction of contact-immunoglobulin.

The boy of 12 years old had acute respiratory disease for 5 days. Local pediatrician detect an arrhythmia at examination of this child. At auscultation: weak first cardiac sound on apex short localized systolic hum in 5 point. On an electrocardiogram - extrasystole, depression of amplitude of T wave. What is the most possible diagnosis? *Infection-allergic myocarditis. Rheumatic myocarditis. Vegetovascular dystonia. Bacterial endocarditis. Functional extrasystole.

The boy of 5 years old who receives ampicillin for treatment of ARD, for 5th day of treatment signs of intoxication had rised, faces became oedematous and joint pain appeared. On skin of a trunk we can see miliary rash. BP-140/90 mm hg. In next day he had excrete 2100 ml of urine (had received 2000 ml of liquids ). In the biochemical analysis of blood: creatinine- 0,22 mmol/L, urea- 11,8 mmol/L, potassium- 3,8 mmol/L, sodium -125 mmol/L. In the analysis of urine: proteinuria- 0,99 g/l, erythrocyturia, eosinophilic leukocyturia. Relative density of urine during the day is 1002 - 1010. What is the most possible diagnosis? *Acute interstitial nephritis. Acute pyelonephritis. Acute glomerulonephritis with nephritic syndrome. Acute glomerulonephritis with nephrotic syndrome. Dysmetabolic nephropathy.

The child of 10 years old complains of reddening of eyes, lacrimation, moderate purulent discharges from conjunctival cavity, sensation of foreign body in eyes. Objectively: hyperemia of palpebral conjunctiva. There is conjunctival injection on eye-bulbes. A cornea is transparent. Pupils are 3 mm in diameter and have quick light response. The lens and vitreous are transparent. An eyeground is in norm. What is the most probable diagnosis? Allergic conjunctivitis. Acute iridocyclitis. *Acute bacterial conjunctivitis. Honoblenorrhea. Adenoviral keratoconjunctivitis.

51 Vaccination

A child born of normal birth. In the mother stated HbsAg carrier. What is the tactics of a doctor relatively to child's immunizations against hepatitis B? *Vaccination against hepatitis B in the first 12 hours of life. Hepatitis B vaccination be postponed for 1 month. Hepatitis B vaccination be postponed for 6 months. Hepatitis B vaccination be postponed for 1 year. Vaccination against hepatitis B is not carried.

Neonate is 5 days old. What vaccination dose of bacillius Calmette-Guerin (bCG) vaccine is necessary for this child? 0.1 Mg. 0.025 Mg. 0.075 Mg. *0.05 Mg. 0.2 Mg.

A neonate is 5 days old. What vaccination dose of BCG vaccine (in mg) is necessary for vaccination of this child? *0,05 mg 0,025 mg 0,075 mg 0,1 mg 0,2 mg

In the inhabited locality there is an increase of diphtheria during the last 3 years with separate outbursts in families. What measure can effectively influence the epidemic process of diphtheria and decrease the morbidity with diphtheria to single cases? Revelation of carriers. *Immunization of the population. Early diagnostics. Hospitalization of patients. Disinfection in disease focus.

The boy of 6 years, fell ill with measles. In the family there is another child of 8 months. What basic counterepidemic measure must be taken relative to contact child? Carry-active immunization against measles. Assign antivirals. *Inject 3 ml of normal human immunoglobulin. Hospitalized in the hospital. Antibiotics.

On the second day after preventive vaccination a 2-year-old boy presented with abdominal pain without clear localization, body temperature rose up to 38ºC. On the third day the child got red papular haemorrhagic eruption on the extensor surfaces of limbs and around the joints. Knee joints were edematic and slightly painful. Examination of other organs and systems revealed no pathological changes. What is the most likely diagnosis? *Haemorrhagic vasculitis Thrombocytopenic purpura Meningococcemia Urticaria DIC syndrome

52 A 12 year old girl complains about abrupt weakness, nausea, dizziness, vision impairment. The day before she ate home-made stockfish, beef. Examination revealed skin pallor, a scratch on the left knee, dryness of mucous membranes of oral pharynx, bilateral ptosis, mydriatic pupils. The girl is unable to read a simple text (mist over the eyes). What therapy would be the most adequate in this case? *Parenteral introduction of polyvalent antibotulinic serum Parenteral disintoxication Parenteral introduction of antibiotics Gastric lavage Parenteral introduction of antitetanus serum

The boy of 4 months old in 15 mines after the second inoculation by diphtheria and tetanus toxoids and pertussis vaccine had signs of a Quincke's edema. What preparation should be used for emergency action? Adrenaline. Furosemide. Heparin. *Prednisolone. Seduxen.

Child 2 years old, sick for 2 weeks, coughing within 7 days, the temperature was subfebrile. Then the coughing attacks of the character got to facial flushing, reprises, a discharge of a viscous, glassy phlegm up to 15 times a day. Hemorrhages in the sclera of both eyes, bridle language are sore. In the lungs there are severe breathing. The introduction of what drug can prevent this disease? *DTP vaccine. Immunoglobulin. Antibiotic. Sulfanilamide. Bacteriophage.

15 years old schoolboy, roller-skate in the court of school, fell and wounded the skin in the section of knee joint. Last inoculation by ADCS to him was made at the age of 5 years. What immunization must be carried out? *Tetanum anatoxin. ADCT. ADT. ADT- M. Antitetanus serum.

At boy of 10 months 15 minutes after the second inoculation by vaccine ACDT fixed the signs of Quincke's edema. What preparation you do use for the rendering to the child of pressing aid? *Prednisolone. Heparin. Papaverine. Curantil. Seduxen.

A child aged 10 months to reduce the temperature paracetamol syrup was given, after 2 hours the temperature was not decreased, and mother gave the child an aspirin at a dose of 0.1 g. After 20 minutes, at child appeared face edema, papular rash, shortness of breath. From history we know that the boy's grandfather did not tolerate aspirin. If this is Quincke's edema, it is the fact that necessary to appoint to child first of all?

53 *Glucocorticoids + antihistamine in / v. Analgin-in / m. Eufillina-in / v. Cardiac glycosides. Antihistamines-enterally.

In child 1 of year with fibroelastose against the background acute respiratory disease suddenly appeared the uneasiness, acrocyanosis, pulse 132 in1 min, respiratory rate 50 into 1 min, bubbling moist rales in the lower divisions, RO2 of 60 mm Hg, pO2 60 mm Hg, pCO2 55 mm Hg At X-ray:- cardiomegaly, strengthening of pulmonary figure, roots is in the form butterfly wings. Worsening in the state is connected with: *Pulmonary edema. Bronchiolitis. Bilateral pneumonia. Angioedema. Lung abscesses.

A 16-year-old adolescent was vaccinated with DTP. In eight days there was stiffness and pain in the joints, subfebrile temperature, urticarial skin eruption, enlargement of inguinal, cervical lymph nodes and spleen. What kind of allergic reaction is observed? *Immunocomplex. Hypersensitivity of delayed type. Cytoxic. Hypersensitivity of immediate type.

HIV AIDS

In boy from the asocial family of the patient with hemophilia of against the background of generalized lymphoadenopathia and recurrent candidiasis of oral cavity developed pneumocystic pneumonia, the correlation coefficient of T- helpers to T-supressors was substantially reduced. What is most possible the reason for the demonstration of opportunistic infections in this patient? The chronic disease of the system of the organs of the digestion. Transitory immunosuppression dependent on age. Hemophilia. *VIH- infection, AIDS. Social-welfare problems

By what way cannot occur the transmission of HIV from mother to child? During pregnancy During delivery Through breast milk *Aerial-droplet way All answers are right.

Passively acquired maternal HIV antibodies may be present in the child’s blood until: A. 6 months of age 12 months of age 16 months of age *18 months of age

54 24 months of age

Major pulmonary illnesses in children with AIDS: Streptococcal pneumonia Bronchial asthma *Pneumocystis pneumonia Adenoviral pneumonitis Emphysema

Major gastro-intestinal illnesses in children with AIDS: Rotaviral gastroenteritis *Unexplained chronic diarrhea Acute duodenal ulcer Phlegmonous pancreatitis Constipation

The HIV refers to the family: Picirnoviridae Herpesviridae *Retroviridae Enteroviridae Paramixoviridae

An antiretroviral drug is: *Lamivudine Rimantadine Aciclovir Ribaverini Zanamivir

The source of infection of AIDS is: *An ill patient An ill animal An ill patient and ill animal Insect Is unknown

The main in pathogenesis of AIDS is: Depression of T-cells Depression of B-cells Stimulation of humoral immunity Depression of thrombocyc *All answers are right.

Congenital AIDS includes: Growth retardation Recurrent bacterial infections Neurological dysfunction Dysmorphic syndrome *All answers are right

The patient of 17 years old complains of liquid excrements during 2 months, body weight loss on 13 kg, weakness, constant subfebrile temperature, and recurrent herpes. Objectively: herpetic rashes on

55 lips, generalized lymphadenopathy, enlargement of liver up to 2 sm. In blood test: red blood cells - 4,4х10/12/l, leucocytes - 10,0х10/9/l, e.-2 %, b.-6 %, s.-61 %, l.-17 %, m.-3 %. atypical lymphocytes - 6 %. What is the most probable diagnosis? Infectious mononucleosis. *AIDS. Lymphogranulomatosis. Amebiasis. Prolonged dysentery.

Pneumonia of pneumocystic etiology, average gravity, RI of 2 stage is diagnosed in 3 monthly child. From what preparation it is necessary to begin treatment? *Biseptol. Penicillin. Cefalexin. Lincomycin. Erythromycin.

The child at the age of 2 weeks, who was born with a gestational age 34-35 weeks was diagnosed generalized candidiasis. Which of these drugs is the preparation of choice for causal treatment? Nystatin. Levorin. Metronidazole. *Diflucan. Klaforan.

TORCH-infections

What infection belongs to the group TORCH-infections? Lues *Herpetic infection Measles Tuberculosis Scarlatina

What variant of disease in child is the most probably if Toxoplasma infection happened in the first trimester of pregnancy? *Miscarriage Mononucleosis- like syndrome Asymptomatic infection Classical manifestation of acute disease Mental retardation

In which period of pregnancy is Toxoplasma infection most dangerous for fetus? *First trimester Latter half Third trimester Second trimester Intranatal

56 What animals are the basic sources of Toxoplasma infection? Pigs Poultry *Cats Freshwater fish Deer

In which period of pregnancy does Toxoplasma infection have minimal risk of affection of fetus? First trimester Latter half *Third trimester Second trimester Intranatal

What variant of disease of children is the most probable if Toxoplasma infection happened in the third trimester of pregnancy? Miscarriage Mononucleosis- like syndrome Classical manifestation of acute disease *Asymptomatic infection Mental retardation

Most frequent form of CMV infection: *Latent Encephalitis Trombocitopenia Hypotrophy Chronic infection

A child of 3 years old has been in hospital for 10 days for herpetic gingivostomatitis. Today the child’s condition is satisfactory, he does not have a fever, eats well, the mucous membrane of the fauces is pink and clean. Erosions are epithelized. When can the child attend children's group? In of 2 weeks after discharge *After clinical recovery In 22 days after the onset of the disease In 7 days after discharge In 12 days after the onset of the disease

A child of 5 years old suffering from allergodermatosis had elevated temperature up to 39.5°С, flaccidity, vomiting; plentiful vesicular rash developed at the sites affected by dermatosis, it was filled with transparent content. Enlarged regional lymph nodes were palpated. In 2 days vesicular content became turbid, umbilicate impression appeared in the centre. The child’s mother suffers from recurrent herpes of the skin. What disease should we think of? Neurodermitis *Caposi herpetiform eczema Zoster Stevens-Johnson's syndrome Chicken-pox

The typical form of localized congenital herpes simplex infection: Liver impairment Renal failure 57 *Skin and mucus lesions Encephalitis Meningitis

The most frequent form of CNS herpes simplex infection in children is Meningitis *Encefalitis Neurotoxicosis Encephalopathy Polyneuritis

What antibiotics do you prefer at the treatment of neonatal sepsis (empirical therapy)? Penicillin + gentamycin. Cefazolin + Ampicillin. Fortum + Amikacin. *Zinacef + Netromycin. Norfloxacin + Metragil.

Sick child who had typhoid, on 14-th day of diseases has suddenly feel stomachache. The child is in consciousness. Pain is acute. The belly takes part in the act of respiration, is not sweel. On the palpation: the moderate muscle tension of a forward abdominal wall, it is more pain in the right inguinal region. Hepatic dullness is not determined. The peristalsis is auscultated, but it's flaccid. Blumberg's sign is positive. Body temperature is 38,9°С. Pulse - 104 impacts in a minute. What is the most probable diagnosis? Perforation of an acute gastric ulcer. Adhesive intestinal obstruction. Acute pancreatitis. Acute perforated appendicitis. *Perforation of a typhoid ulcer.

The flash of food poisoning was registered in urban village. The diagnosis of botulism was put on the basis of a clinical picture of disease. What of the listed products are necessary for selecting, first of all, on the analysis for verification of diagnosis? *Canned food. Pasteurized milk. Cabbage. Boiled meat. Potato.

The girl of 12 years old complains of weakness, nausea, giddiness, disturbance of vision. She ate home-brewed dried fish and beef on the day before. At examination we can see the pale skin, scratch of the left knee, dryness of guttur's mucosa, bilateral ptosis, pupils are dilated. Girl is not unable to read a simple text ("grid", "fog" before eyes). What therapy will be the most adequate in this case? *Parenteral administration of polyvalent anti-botulism serum. Parenteral deintoxication. Parenteral administration of antibiotics. Gastric lavage.

58 Parenteral administration of antitetanic serum.

The purulent discharges from the umbilical wound of newborn are marked; a skin around the navel is tumescent .Objectively: pallid skin with a yellow-grey tint, generalized hemorrhagic rash. The body temperature is hectic. What is the most probable diagnosis? *Sepsis. Hemorrhagic disease of newborn. Thrombocytopathy. Omphalitis. Hemolytic disease of newborns.

The child of 12 years old was hospitalized with an intestinal obstruction. During operation it was found out that obstruction of small bowel was caused by ball of worms. What worm was it? What it was a helminth? Cysticercus. *Ascarid. Filaria. Pinworm. Hymenolepis nana.

Determine the preparation, which does have a effectiveness for treatment enterobioses and ascariasis in children and is assigned single-time: *Pyrantel. Vermoks. Piperazine adininat. Naftamon. Ditianazin.

What preparation is effective for treatment of enterobiasis and ascaridiasis of children and it is prescribe for a one taking? *Pyrantel. Vermox. Pyperasini adinatis. Naftamon. Ditianasin.

Sick 13 years old boy complaint of a pain and edema of ankle joints, gripes and frequent urination, reddening of a conjunctiva of eyes. What is your preliminary diagnosis? *Reiter's disease. Juvenile pseudorheumatism. Infectious-allergic polyarthritis. Systemic lupus erythematosus.

59

Recommended publications